You are on page 1of 266

1.

A 37-year-old patient presented to you with


hyper-extension of 4th and 5th
metacarpophalangeal joint with flexion at
proximal interphalangeal joint. This
deformity is due to injury to:
a) Deep branch of ulnar nerve

b) Median nerve

c) Radial nerve

d) Superficial branch of median nerve

Correct Answer - A
Ans: A. Deep branch of ulnar nerve
(Ref: Gray's 41/C p784, 866, 40/e p888)
Deformity = Claw hand.
Due to injury of deep branch of ulnar nerve.

Available in the Channel


t.me/mmedicalbooks
click on the box for the Channel link
2. Which of the following nerves supplies the
ear lobule?
a) Greater auricular nerve

b) Lesser occipital nerve

c) Facial nerve d

d) Auriculotemporal nerve

Correct Answer - A
Ans:. A. Greater auricular nerve
(Ref Gray's 41/e p627, 40/e p620)
Greater auricular nerve supplies ear lobule.
Sensory nerves of ear:
Great auricular nerve - Supplies most of cranial surface & posterior
part of lateral surface (helix, antihelix, lobule).
Lesser occipital nerve - Supplies upper part of cranial surface.
Auricular branch of vagus - Supplies concavity of concha &
posterior part of eminentia.
Auriculotemporal nerve - Supplies tragus, crus of helix & adjacent
part of helix.
Facial nerve + auricular branch of vagus - Supplies small areas
on both aspects of auricle, concha depression & its eminence.
Join Telegram Channel
@mmedicalbooks
for all NEET PG /JIPMER/PGI/FMGE /AIIMS Questions papers with
answers and explainations and
All kinds of TEXTBOOKS & NOTES Free
Available in the Channel
t.me/mmedicalbooks
3. Which of the following structures are not
involved in development of diaphragm?
a) Somatic body wall

b) Septum transversum

c) Pleuroperitoneal membrane

d) Pleuropericardial membrane

Correct Answer - D
Ans: D. Pleuropericardial membrane
Pleuropericardial membrane:
Supradiaphragmatic structure.
Not involved in formation of diaphragm.
Development of Diaphragm:
Septum transversum - Central tendon.
Pleuroperitoneal membranes → Small intermediate muscular
portione.
Mesentery of esophagus → Crurae.
Body wall → Peripheral muscular diaphragm.
Cervical myotomes (muscular input).

Join Telegram Channel


@mmedicalbooks
for all NEET PG /JIPMER/PGI/FMGE /AIIMS Questions papers with
answers and explainations and
All kinds of TEXTBOOKS & NOTES Free
Available in the Channel
t.me/mmedicalbooks
4. Which of these is an immune-privileged
site?
a) Area postrema

b) Loop of Henle

c) Optic nerve

d) Seminiferous tubules

Correct Answer - D
Ans: D. Seminiferous tubules
Ref Grays 40/e p52; Robbins 9/e p214; Ganong 25/e p418, 24/e
p420)
Seminiferous tubule of testis - Immune privileged site.
Immune privileged site:
Hidden (sequestered) antigens formed only in where there is no
communicate between blood & lymph.
Hence, self-antigens in these tissues fail to elicit immune responses
à Essentially ignored by immune system.
Immune- Privileged Sites
Placenta & fetus°
Testis°
Brain°
Eye (Anterior camber)°
Join Telegram Channel
@mmedicalbooks
for all NEET PG /JIPMER/PGI/FMGE /AIIMS Questions papers with
answers and explainations and
All kinds of TEXTBOOKS & NOTES Free
Available in the Channel
t.me/mmedicalbooks
5. Craniovertebral joint does not include:
a) Occipital condyle

b) Axis

c) Atlas

d) Wings of sphenoid

Correct Answer - D
Ans: D. Wings of sphenoid
(Ref Gray's 40/e p733)
Craniovertebral Joint:
Consists of occipital condyles, atlas & axis.
Articulation between cranium & vertebral column specialized – To
provide a wider range of movement.
Functions as a universal joint.
Permits horizontal & vertical scanning movements of head.
Adapted for eye-head co-ordination.
Join Telegram Channel
@mmedicalbooks
for all NEET PG /JIPMER/PGI/FMGE /AIIMS Questions papers with
answers and explainations and
All kinds of TEXTBOOKS & NOTES Free
Available in the Channel
t.me/mmedicalbooks
6. Pronator teres syndrome is related to
which nerve ?
a) Radial

b) Median

c) Ulnar

d) Axillary

Correct Answer - B
Ans. is 'b' i.e., Median

Join Telegram Channel


@mmedicalbooks
for all NEET PG /JIPMER/PGI/FMGE /AIIMS Questions papers with
answers and explainations and
All kinds of TEXTBOOKS & NOTES Free
Available in the Channel
t.me/mmedicalbooks
7. Maxillary bone does not articulate with:
a) Ethmoid

b) Sphenoid

c) Frontal

d) Lacrimal

Correct Answer - B
Ans: B. Sphenoid
(Ref Gray's 41/e p484, 40/e p473-476)
Maxillary bone does not articulate with sphenoid.
Articulation of maxilla:
Each maxilla articulates with nine bones:
Two of cranium: Frontal & ethmoid.
Seven of the face: Nasal, zygomatic, lacrimal, inferior nasal
concha, palatine, vomer & adjacent fused maxilla.
Sometimes articulates with orbital surface & with lateral pterygoid
plate of sphenoid.

Join Telegram Channel


@mmedicalbooks
for all NEET PG /JIPMER/PGI/FMGE /AIIMS Questions papers with
answers and explainations and
All kinds of TEXTBOOKS & NOTES Free
Available in the Channel
t.me/mmedicalbooks
8. True statements about osteoblasts are all
except:
a) Derived from osteoprogenitor cells

b) Regulated by BM P

c) Have a plasma membrane showing multiple folds

d) Have neuropeptide receptors

Correct Answer - C
Ans: C. Have a plasma membrane showing multiple folds
(Ref Gray's 40/e p87, 88, 91)
Characteristic of osteoclasts:
Plasma membrane showing multiple folds.
I.e. ruffled borders are involved in osteocytic resorption.
Osteoclasts:
Large (40 or more) polymorphic cells.
Have upto 20 oval, closely packed nuclei.
Howship's lacunae - Lie in close contact with bone surface in
resorption bays.
Cytoplasm:
Contains numerous coated transport vesicles & microtubule arrays.
Involved in vesicles transport between Golgi stacks & ruffled
membrane.
Ruffled membrane - Highly infolded cell surface of active osteoclasts
at local bone resorption sites.
Osteoblasts:
Derived from osteoprogenitor (stem) cells of mesenchymal origin.
Present in bone marrow & other connective tissues.
Proliferate & differentiate after stimulated by bone morphogenetic
proteins (BMPs à into osteoblasts prior to bone formation.
Join Telegram Channel
@mmedicalbooks
for all NEET PG /JIPMER/PGI/FMGE /AIIMS Questions papers with
answers and explainations and
All kinds of TEXTBOOKS & NOTES Free
Available in the Channel
t.me/mmedicalbooks
9. Which of the following vessels supply the
anal canal?
a) Superior rectal artery

b) Middle rectal artery

c) Inferior rectal artery

d) all of the following

Correct Answer - D
Ans: D. all of the above arteries supply rectal canal
(Ref: Gray's 41/e p1058, 40/e p1155-1159; Manual of Total
Mesorectal Excision by Moran, Richard John Heald (2013)/p40)
Anal canal – Supply:
By superior rectal, middle rectal, inferior rectal branches of pudendal
artery.
Occasionally by median sacral artery.

Join Telegram Channel


@mmedicalbooks
for all NEET PG /JIPMER/PGI/FMGE /AIIMS Questions papers with
answers and explainations and
All kinds of TEXTBOOKS & NOTES Free
Available in the Channel
t.me/mmedicalbooks
Join Telegram Channel
@mmedicalbooks
for all NEET PG /JIPMER/PGI/FMGE /AIIMS Questions papers with
answers and explainations and
All kinds of TEXTBOOKS & NOTES Free
Available in the Channel
t.me/mmedicalbooks
10. Buccinator is pierced by all of the
following except:
a) Labial branch of facial nerve

b) Buccal branch of mandibular nerve

c) Parotid duct

d) Molar mucous glands

Correct Answer - B
Ans: B. Buccal branch of mandibular nerve
(Ref Grays 41/e 4(Y 10/e p487; Snells 9/e p582)
Buccal branch of facial run crosses the buccinator muscle and
innervates it, without piercing it.
Buccinator - Structures passing:
Anteriorly (Superficial buccinator surface):
Muscles - zygomaticus major, risorius, levator & depressor anguli
oris muscles.
Duct - Parotid duct.
Pierces buccinator opposite third upper molar tooth & lies on deep
muscle surface before opening into mouth opposite maxillary second
molar tooth.
Join Telegram Channel
Blood vessel - Crossed by facial artery, facial vein & branches of
facial & buccal nerves.
@mmedicalbooks
for all NEET PG /JIPMER/PGI/FMGE /AIIMS Questions papers with
answers and explainations and
All kinds of TEXTBOOKS & NOTES Free
Available in the Channel
t.me/mmedicalbooks
11. All of the following are true about location
of otic ganglia except:
a) Inferior to foramen ovale

b) Lateral to tensor veli palatini

c) Lateral to mandibular nerve

d) Anterior to middle meningeal artery

Correct Answer - C
Ans: C. Lateral to mandibular nerve
(Ref Gray 41/e p552. 40/e p543)
Mandibular nerve lies lateral to otic ganglion.
Ie., Ganglion lies medial to mandibular nerve.
Otic ganglion:
Small, oval, flat reddish-grey ganglion.
Situated just below foramen ovate.
Peripheral parasympathetic ganglion located in the infratemporal
fosse.
Functionally associated with glossopharyngeal nerve & innervates
parotid gland for salivation.
Connected to chorda tympani nerve & to nerve of pterygoid canal.
Pathways provide an alternate pathway of taste from anterior two-
Join Telegram Channel
thirds of tongue.
@mmedicalbooks
for all NEET PG /JIPMER/PGI/FMGE /AIIMS Questions papers with
answers and explainations and
All kinds of TEXTBOOKS & NOTES Free
Available in the Channel
t.me/mmedicalbooks
12. Calculate the ejection fraction from the
given volume pressure curve:
a) 40%

b) 50%

c) 55%

d) 60%

Correct Answer - D
Ans: D. 60%
(Ref Gunong 25/e p540 24/e p542)
Ejection fraction calculated from the given volume pressure curve is
60%.

Pressure Volume Loop:


ab: isovolumetric contraction
bc: Ventricular contraction during systole
cd: Isovolumetric relaxation.
Calculation:
End-Diastolic Volume (EDV) (Point a) = 130 mL
End Systolic Volume (ESV) (Point d) = 50 mL
Stroke Volume (SV) = EDV–ESV = 80 mL
Ejection Fraction = SV/EDV = 80/130 = 0.6 i.e. % EF = 60%.

Join Telegram Channel


@mmedicalbooks
for all NEET PG /JIPMER/PGI/FMGE /AIIMS Questions papers with
answers and explainations and
All kinds of TEXTBOOKS & NOTES Free
Available in the Channel
t.me/mmedicalbooks
13. Sodium iodide symporter is not present in
a) Pituitary gland

b) Placenta

c) Parotid

d) Thyroid

Correct Answer - A
Ans: A. Pituitary gland

Sodium iodide symporter - Not present in the pituitary gland.
Sodium-Iodide Symporter (NIS) is present in
Thyroid° Ciliary body of the eye°
Salivary glands° Choroid plexus°
Gastric mucosa° Mammary glands°
Placentaο Certain cancers derived from these

Join Telegram Channel


@mmedicalbooks
for all NEET PG /JIPMER/PGI/FMGE /AIIMS Questions papers with
answers and explainations and
All kinds of TEXTBOOKS & NOTES Free
Available in the Channel
t.me/mmedicalbooks
14. Interstitial fluid volume can be determined
by:
a) Radioactive iodine and radiolabelled water

b) Radioactive water and radiolabelled albumin

c) Radioactive sodium and radioactive water

d) Radioactive sodium and radioactive labelled albumin

Correct Answer - D
Ans: D. Radioactive sodium and radioactive labelled albumin
(Ref Ganong 25/e p3, 24/e p2; Guvton I p309, 310)
Fluid volume Indicator used
Total body Deuterium oxide: D20 (MC used)°
water volume Tritium oxide°, Aminopyrine°, Antipyrine°
Extracellular Inulin (most accurate)°s
fluid volume 22Na, 1251-iothalamate, thiosulfate°
Evans blue°
Plasma volume Serum albumin labeled with radioactive iodine° (1-
125)
Intracellular (Calculated as total body water - extracellular fluid
fluid volume)
Join Telegram Channel
"Cr-labeled RBCs°, or calculated as blood
@mmedicalbooks
Blood volume
volume = plasma volume/ (1 - hematocrit)
for all NEET (Calculated as extracellular fluid volume - plasma
PG /JIPMER/PGI/FMGE /AIIMS Questions papers with
Interstitial fluid
answers andvolume)explainations and
All kinds of TEXTBOOKS & NOTES Free
Available in the Channel
t.me/mmedicalbooks
15. Considering the latent period of muscle
twitch to be 10 ms, contraction time 40 ms
and relaxation time 50 ms, what will be the
tetanizing frequency for this muscle?
a) 25 Hz

b) 50 Hz

c) 100 Hz

d) 75 Hz

Correct Answer - A
Ans: A. 25 Hz
Stimulation frequency at which summation of contractions occurs –
Determined by twitch duration of the particular muscle under study.
Example:
If twitch duration = 10 ms.
Frequencies less than 1/10 ms (100/s) - Cause discrete responses
interrupted by complete relaxation.
Frequencies greater than 100/s cause summation.
Tetany:
Continuous contraction of muscle fibers, without latent period &
relaxation time.
Hence tetanizing frequency depends only on contraction time, i.e.
twitch duration.
Twitch duration = 40 milliseconds = 0.04 sec
Tetanizing frequency = 1/0.04 = 25 Hz
16. All of these are actions of Atrial
Natriuretic Peptide except:
a) Afferent arteriole dilation

b) Mesangial constriction

c) Decreased sodium absorption in PCT

d) Inhibition of sodium reabsorption in medullary collecting duct

Correct Answer - B
Ans: B. Mesangial constriction
(Ref Ganong 25/e p705, 24/e p707).
Actions of Natriuretic Hormones:
Renal actions:
Increases renal Na+ excretion à Increase glomerular filtration.
By dilating afferent arterioles & relaxing mesangial cells.
Mainly circulating ANP & BNP.
Act on renal tubules to inhibit Na+ reabsorption.
Inhibits renin secretion.
Vascular effects:
Increases capillary permeability à Fluid extravasation à reduced BP.
Relax vascular smooth muscle in arterioles & venules.
CNP - Greater dilator effect on veins than ANP and BNP.
Hormonal effects:
Counteracts pressor effects of catecholamines & angiotensin II.
17. The extracellular potassium concentration
is 100 mEq/mmol and intracellular
potassium concentration is 10 mEq/mmol.
What will be the equilibrium potential for
potassium according to Nernst equation?
a) 0 V

b) —60 V

c) —90 V

d) +30 V

Correct Answer - B
Ans: B. —60 V
Equilibrium potential:
Membrane potential at which equilibrium exists between influx &
efflux of ions.
Nernst equation:
Physiological application for calculating ion potential of charge z
across membrane.
Determined using ion concentration both inside & outside cell.
Concentration (mmol/L of 1120) of some ions inside and
outside mammalian motor neurons
Ion Inside cell Outside cell Equilibrium potential (my)
Na+ 15.0 150.0 +60
K+ 150.0 5.5 –90
Cl- 9.0 125.0 –70
18. In the formula for urea clearance, C = U x
V/P, what does U stands for:
a) Urinary concentration in g/24 hour

b) Urinary concentration in mg/ml

c) Urine osmolarity

d) Urine volume per minute

Correct Answer - B
Ans: B. Urinary concentration in mg/ml
(Ref: Ganong 25/p p676. 21/c p678)
In given formula for measuring GFR:
C = Clearance of the substance
U = Urinary concentration of the substance in mg/ml
P = Plasma concentration of the substance in mg/ml
V = Volume of urine

Join Telegram Channel


@mmedicalbooks
for all NEET PG /JIPMER/PGI/FMGE /AIIMS Questions papers with
answers and explainations and
All kinds of TEXTBOOKS & NOTES Free
Available in the Channel
t.me/mmedicalbooks
19. Which of the following methods is used
for calculation of anatomical dead space?
a) Xenon dilution technique

b) Bohler's method

c) Spirometry

d) Single breath nitrogen test

Correct Answer - D
Ans: D. Single breath nitrogen test
(Ref: Ganong 25/e p633, 634, 24/e p633, 634)
Anatomical dead space – Calculation:
By Bohr's equation - Uses single breath nitrogen inhalation
technique.
Xenon/Helium dilution technique:
Used to measure functional residual capacity of lung.
Spirometry:
Cannot measure residual or dead space volumes.

Join Telegram Channel


@mmedicalbooks
for all NEET PG /JIPMER/PGI/FMGE /AIIMS Questions papers with
answers and explainations and
All kinds of TEXTBOOKS & NOTES Free
Available in the Channel
t.me/mmedicalbooks
20. Which of these is not a cofactor for
glycogen phosphorylase, an important
enzyme of the glycogenolysis pathway?
a) Calmodulin

b) c-AMP

c) Protein Kinase A

d) Glycogenin

Correct Answer - D
Ans: D. Glycogenin
(Ref Harper 30/c p 181)
Glycogenin:
An enzyme involved in glucose conversion to glycogen.
Acts as a primer - By polymerizing first few glucose molecules à
enzymes take over.
Involved in glycogen synthesis pathway rather than glycogenolysis.

Join Telegram Channel


@mmedicalbooks
for all NEET PG /JIPMER/PGI/FMGE /AIIMS Questions papers with
answers and explainations and
All kinds of TEXTBOOKS & NOTES Free
Available in the Channel
t.me/mmedicalbooks
21. After a point mutation, glutamic acid is
replaced by valine, which leads to
formation of sickle cell hemoglobin. The
mobility of HbS as compared with normal
hemoglobin on gel electrophoresis will
be:
a) Decreased

b) Increased

c) Dependent on HbS concentration

d) Unchanged.

Correct Answer - A
HbS mobility on gel electrophoresis:
Decreased mobility - Compared to normal hemoglobin.
Electrophoresis of hemoglobin:
Obtained from lysed red blood cells.
Used in sickle cell trait & disease diagnosis.
Sequence of Movement: HbA2 < HbC < HhS < HbF < HbA
@mmedicalbooks
for all NEET PG /JIPMER/PGI/FMGE /AIIMS Questions papers with
answers and explainations and
All kinds of TEXTBOOKS & NOTES Free
Available in the Channel
t.me/mmedicalbooks
22. Thiamine is a cofactor for all of the
following enzymes except:
a) Alpha ketoglutarate dehydrogenase

b) Branched-chain keto-acid dehydrogenase

c) Succinate dehydrogenase

d) Pyruvate dehydrogenase.

Correct Answer - C
Ans: C. Succinate dehydrogenase
Thiamin as coenzyme:
Catalyzes oxidative decarboxylation reactions.
3 multi-enzyme complexes catalyzing oxidative decarboxylation
reactions:
Branched-chain ketoacid dehydrogenase - Involved in metabolism of
leucine, isoleucine & valine
Alpha-ketoglutarate dehydrogenase – In citric acid cycle
Pyruvate dehydrogenase - In carbohydrate metabolism
Transketolase reaction – In pentose phosphate pathway.
Succinate dehydrogenase:
Involved in redox reaction catalyzed by FMN & FAD.
23. The cofactor vitamin B12 is required for
the following conversion:
a) Dopamine to Norepinephrine

b) Propionyl CoA to methyl malonyl CoA

c) Methyl malonyl CoA to succinyl CoA

d) Cysteine to homocysteine

Correct Answer - C
Ans: C. Methyl malonyl CoA to succinyl CoA
(Ref: Harper 30/e p550, 558, 28ie p346)
Vitamin B12 as Cofactor for:
Methylmalonyl CoA mutase - Isomerization of methylmalonyl co-A to
succinyl co-A.
Methionine synthase - Methylation of pyrimidine ring to form
thymine.
Homocysteine methyl transferase - Methylation of homocysteine to
methionine
Metabolism of diol.
In bacteria for interconversion of glutamate & beta-methyl aspartate°

Join Telegram Channel


@mmedicalbooks
for all NEET PG /JIPMER/PGI/FMGE /AIIMS Questions papers with
answers and explainations and
All kinds of TEXTBOOKS & NOTES Free
Available in the Channel
t.me/mmedicalbooks
24. Glycogen synthesis and breakdown takes
place in the same cell, having enzymes
necessary for both the pathways. Why the
glucose-6-phosphate, freshly synthesized
during glycogenesis in cytoplasm of
hepatocytes, is not immediately degraded
by the enzyme glucose¬6-phosphatase?
a) The thermodynamics does not favor such a reaction to occur

b) Glucose-6-phosphatase is present in the endoplasmic reticulum


and cannot act on glycogen formed in the cytoplasm

c) Glycogenesis and glycogenolysis are tightly regulated such that


enzymes of only one of those is present at a time.

d) Steric hindrance due to albumin

Correct Answer - B
Ans: B. Glucose-6-phosphatase is present in the endoplasmic
reticulum and cannot act on glycogen formed in the cytoplasm
(Ref Harper 30/e p178)
Glucose-6-phosphate:
Formed in cytoplasm of hepatocytes.
Glucose-6-phosphatase:
Present in lumen of smooth endoplasmic reticulum of cell.
Reaction does not take place until gluconeogenesis is favored.
Glucose-6-phosphatase catalyzes hydrolysis of glucose-6-
phosphate in liver → Exports glucose à increases blood glucose
concentration
25. In lead poisoning, there is an inhibition of
some of the enzymes of the heme
biosynthetic pathway. This is reflected by
the accumulation of what substance in
blood?
a) Uroporphyrinogen III

b) Ferrochelatase

c) Porphob I inogen

d) Delta amino levulinic acid

Correct Answer - D
Ans: D. Delta amino levulinic acid
(Ref: Harper 30/e n329)
Lead poisoning:
Affect heme metabolism by combining with SH groups in
ferrochelatase & ALA (delta-amino levulinic acid) dehydratase
enzymes.
Elevated protoporphyrin levels - Found in RBC’s.
Elevated ALA & coproporphyrin levels - Found in urine.
26. Sites of heme synthesis are all of these
except:
a) RBC

b) Hepatocytes

c) Osteocytes

d) Bone marrow

Correct Answer - A
Ans: A. RBC
(Ref: Harper 30/e p325)
Heme biosynthesis:
Occurs in most mammalian cells except mature erythrocytes.
Due to lacking mitochondria.
85% - In erythroid precursor cells in bone marrow.
15% heme synthesis - In hepatocytes.

Join Telegram Channel


@mmedicalbooks
for all NEET PG /JIPMER/PGI/FMGE /AIIMS Questions papers with
answers and explainations and
All kinds of TEXTBOOKS & NOTES Free
Available in the Channel
t.me/mmedicalbooks
27. What is the codon for selenocysteine?
a) UAG

b) UGA

c) UAA

d) GUA

Correct Answer - B
Ans: B. UGA
(Ref Harper 30/e p18, 286)
Selenocysteine:
Commonly termed "21st amino acid".
Present in biological proteins.
Not coded directly in genetic code.
Instead encoded by a UGA codon.
UGA codon - Normally a stop codon.
This mechanism called as|”translational recoding”.
28. All of these substrates are glucogenic
except:
a) Acetyl CoA

b) Pyruvate

c) Glycerol

d) Lactate

Correct Answer - A
Ans: A. Acetyl CoA
(Ref: Harper 30/e p185. 29/e p187)
Acetyl CoA:
Not substrate for gluconeogenesis.
Cannot be converted back to glucose.
Since acetyl CoA cannot be converted back to pyruvate as carbon
backbone is lost during citric acid cycle as CO2.
Substrates for Gluconeogenesis
Glucogenic amino acids (all except Leucine Pyruvate
& lysine which are purely ketogenic): Most important is Propionate
alanine Glycerol
Lactate Fumarate

29. Which of the following conversions does
not require Biotin as a cofactor?
a) Gamma carboxylation of glutamate

b) Acetyl Co-A to Malonyl Co-A

c) Propionyl Co-A to methyl malonyl Co-A

d) Pyruvate to oxaloacetate

Correct Answer - A
Ans: A. Gamma carboxylation of glutamate
(Ref: Harrison 19/e p96e-5; Harper 30/e p550, 561)
Gamma carboxylation of glutamate:
Carried out by Gamma-glutamyl carboxylase.
Gamma-glutamyl carboxylase:
Vitamin-K dependent enzyme.
Catalyzes post-translational modification of vitamin K-dependent
proteins.
Biotin:
Transfers carbon dioxide in reactions like acetyl-CoA carboxylase,
pyruvate carboxylase, propionyl-CoA carboxylase & methylcrotonyl-
CoA carboxylase.
Biotin functions to transfer carbon dioxide in reactions
Pyruvate (3C) to oxaloacetate (4C) in
Pyruvate carboxylase
gluconeogenesis°
Acetyl-CoA (2C) to malonyl-CoA (3C) in
Acetyl-CoA carboxylase
lipid synthesis.
Propionyl-CoA Propionyl-Co A (3C) to methylmalonyl-
carboxylase CoA (4C) in
Methylmalonyl
Propionic acid synthesis in bacteria
Propionic acid synthesis in bacteria
CoA carboxyl
Methylcrotonyl-CoA
Leucine catabolism
carboxylase
30. Which of the following is not a technique
for protein precipitation?
a) Trichloroacetic acid

b) Heat precipitation

c) Isoelectric point method

d) Titration with reducing sugar

Correct Answer - D
Ans: D. Titration with reducing sugar
(Ref Harper 30/e p26)
Titration with reducing sugar is not used for protein precipitation.
Protein precipitation:
Widely used to concentrate proteins & purify them from various
contaminants.
Used for isolating specific protein in quantities sufficient for analysis.
Requires multiple successive purification techniques.
Classic approaches:
Based on,
Isoelectric precipitation - Differences in relative solubility of individual
proteins as pH function.
Precipitation with ethanol or acetone – Polarity.
Salting out with ammonium sulfate.
Paper chromatography including TLCD.
31. Most important but nonspecific regulator
of iron metabolism is:
a) Hepcidin

b) DMT I

c) Ferroportin

d) Ferritin

Correct Answer - A
Ans: A. Hepcidin
(Ref Robbins 91e p650 848 8/e p660)
Hepcidin:
Main regulator protein for iron absorption.
Encoded by HAMP gene.
Small circulating peptide synthesized & released from liver in
response to increased intrahepatic iron levels.
Proteins Involved in Iron Metabolism
Ceruloplasmin° (ferrioxidase activity) HFE°
DMT1° Iron-
Ferrireductase° (cytochrome b reductase Ferroprtin responsive
I) Heme element-
Ferritin° transporter binding
Hemojuvelin protein°
Hepcidin° Transferrin°
Hephaestin° Transferrin
receptors 1
& 2
32. Which of these is the most important
prognostic factor in ALL?
a) Hyperploidy

b) Total leucocyte count greater than 50,000

c) Age

d) Response to steroids

Correct Answer - D
Ans: D. Response to steroids
(Ref Wintrobes Clinical Hematology 12/e p1892; Robbins 9/e 1)590-
593, 8/e p627, 628)
Most important predictive factors in ALL:
Age at diagnosis time.
Initial leukocyte count.
Speed of steroid treatment response:
Most consistent prognostic marker.
Mainly acts by rapidly removing blast cells from marrow or peripheral
blood.
33. In genomic imprinting. DNA is modified
by:
a) Acetylation

b) Methylation

c) Phosphorylation

d) Deamination

Correct Answer - B
Ans: B. Methylation
(Ref Robbins 9/e p180, 8/e pl)
Genomic imprinting:
DNA modified by methylation.
An epigenetic process resulting in differential inactivation of either
maternal or paternal alleles of certain genes.
Mechanism:
DNA methylation at CG nucleotide.
Histone H4 deacetylation.
Methylation.
34. About intraoperative histopathological
analysis, all are true except:
a) Gives an immediate definitive diagnosis of tumor

b) Used for detecting positive margins after resection

c) Used to confirm suspected metastasis

d) Sentinel lymph node biopsy in breast carcinoma is an example

Correct Answer - A
Ans: A. Gives an immediate definitive diagnosis of tumor
(Ref http:/,www.nchi.iilm.nih.gov/pmc/arii,PMC3347896).
lntraoperative Histopathological Analysis:
Frozen section procedure/cryosection:
Rapid microscopic specimen analysis.
Used most often in oncological surgery.
Slide quality produced by frozen section lower than formalin-fixed
paraffin-embedded tissue processing.
Used for presumptive diagnosis.
More accurate diagnosis - Fixed tissue processing.
35. Drugs that should be given with
prescription of registered medical
practitioner only are included in which
schedule ?
a) Schedule C

b) Schedule E

c) Schedule H

d) Schedule I

Correct Answer - C
Ans. is 'c' i.e., Schedule H
36. Which of these drugs is a calcineurin
inhibitor?
a) Cyclosporine

b) Methotrexate

c) Azathioprine

d) Mycophenolate mofetil

Correct Answer - A
Ans: A. Cyclosporine
Cyclosporine:
Calcineurin inhibitor.
MOA:
Targets intracellular signaling pathways induced as consequence of
T cell–receptor activation.
37. Which of the following is an
antifibrinolytic agent?
a) Dabigatran

b) Protamine

c) Alteplase

d) Epsilon aminocaproic acid

Correct Answer - D
Ans: D. Epsilon aminocaproic acid
(Ref Goodman Gilman 12/e p867; Katzung 13/e p599-600, 12/e
p616; KDT 7/e p628, hie p6081
Epsilon-aminocaproic acid:
Synthetic inhibitor of plasmin-plasminogen system.
Only commercially available & potent antifibrinolytic agent.
38. At low infusion rates of 3-5 mcg/kg/min,
what action is produced by dopamine?
a) Vasoconstriction

b) Increased renal blood flow

c) Increased cardiac contractility

d) Decreased blood pressure

Correct Answer - B
Ans: B. Increased renal blood flow
(Ref: Goodman Gilman 12/e p355; KDT 7/e p134. 6/e p507)
Action of dopamine:
At low concentrations (2 to 5 mcg/kg per minute):
Primarily acts on vascular D1 receptors.
Mainly renal, mesenteric & coronary beds.
By activating adenylyl cyclase à raising intracellular cyclic AMP
concentrations.
D1 receptor stimulation à Vasodilation.
Increases in glomerular filtration rate, renal blood flow & Nat+ K+ -
ATPase pump.
39. Platelet aggregation is blocked by aspirin
due to its action on:
a) Prostacyclin

b) PGF 2 alpha

c) Thromboxane A2

d) Phospholipase C

Correct Answer - C
Ans: C. Thromboxane A2
(Ref: Goodman Gilman 12/c n868.- Katzung, 13/c p621. 12/e p638:
KDT 7/e p195, 6/e p186. 60('
Aspirin MOA:
Blocks TxA2 production.
By acetylating a serine residue near active site of platelet
cyclooxygenase-1 (COX-1).
COX-1:
Enzyme producing cyclic endoperoxide precursor of TxA2.
Platelets do not synthesize new proteins → Hence aspirin action on
platelet COX-1 is permanent, lasting.
Until platelet life (7-10 days).
Repeated doses of aspirin produces cumulative effect on platelet
function.
TxA2 (thromboxane A2):
Major cyclooxygenase product in platelets.
Labile inducer of platelet aggregation.
Potent vasoconstrictor.
40. Phase 1 clinical trial is done for:
a) Drug safety

b) Pharmacodynamics

c) Efficacy

d) Dosing

Correct Answer - A
Ans: A. Drug safety
(Ref Goodman Gilman 12/e p79; .Katzung 13/e p12, 12/e p75; KDT
7/e p63-64, 6/e p77)
Safety, pharmacodynamics & dosing - Tested in Phase 1
clinical trial.
Drug safety - Most important primary end point.
41. Which of the following drugs acting on
dilator pupillae has an action analogous
to that of pilocarpine on sphincter
papillae?
a) Timolol

b) Epinephrine

c) Neostigmine

d) Tropicamide

Correct Answer - B
Ans: B. Epinephrine
(Ref Goodman Gilman 12/e p326,. Katzung, 13/e p162, 12/e p160,.
KDT 7th/e p153. 6th/e p123).
Epinephrine acts on dilator pupillae, causing the dilatation of pupil
analogous to that of pilocarpine on sphincter papillae.
Sympathetic stimulation of adrenergic receptors causes the
contraction of the radial muscle and subsequent dilation of the pupil.
Parasympathetic stimulation causes contraction of the circular
muscle and constriction of the pupil.
42. Which of the following drugs does not
affect DNA synthesis?
a) Rifampicin

b) Linezolid

c) Nitrofurantoin

d) Metronidazole

Correct Answer - B
Ans: B. Linezolid
(Ref Goodman Gilman I3/e p796, 12/e p1537; Katzung 12/e p817;
KDT 7/e p758, 817, 6/e p669)
Linezolid inhibits protein synthesis by binding to the P site of the 50S
ribosomal subunit and preventing formation of the larger ribosomal-
fMet-tRNA complex that initiates protein synthesis.
Nitrofurantoin - Works by damaging bacterial DNA.
Metronidazole - Forms toxic free radical metabolites in the bacterial
cell that damage DNA.
Rifampicin - Inhibit DNA-dependent RNA polymerase.
43. What does low volume of distribution of a
drug mean?
a) Low bioavailability

b) Does not accumulates in tissues

c) Low absorption

d) Not metabolized in the body

Correct Answer - B
Ans: B. Does not accumulates in tissues
(Ref: KDT 7/e p17, 18, 6/e p18)
Low volume of distribution:
Implies that the drug remains confined to the plasma compartment
without getting distributed in the body tissues.
44. Which of the following drugs is not used
in treatment of bird flu?
a) Oseltamivir

b) Ribavirin

c) Zanamivir

d) Peramivir

Correct Answer - B
Ans: B. Ribavirin
(Ref Harrison 19/e p1214; Goodman Gilman 12/e p1609, 1615;
Katzung 13/e p861, 12/e p886, 887; KDT 7/e p798, 804, 6/e p777)
Oral ribavirin:
Used for chronic HCV infection, not the bird flu.
Standard treatment for chronic HCV infection - Oral ribavirin
combined with injected pegIFN alfa-2A or -2B.
DOC for bird flu (H5N1).
Oseltamivir Treatment of uncomplicated influenza A or B in
healthy adults.
Administered by inhalational route.
Zanamivir Treatment of uncomplicated influenza A or B in
healthy adults.
Administered IV
Peramivir
Activity against both influenza A & B viruses
Long acting neuraminidase inhibitor
Laninamivir
Effective against oseltamivir resistant virus
Amantadine Active against influenza A only
Longer acting than Amantadine, active
Rimantadine
against influenza A only
45. What are the appropriate instructions to
be given while prescribing
bisphosphonates to a patient:
a) To be given empty stomach with a glass of water

b) Taken along with food

c) Stop if features of gastritis develop

d) Stop if bone pains occur.

Correct Answer - A
Ans: A. To be given empty stomach with a glass of water
(Ref: Gilman 12/e p1296; Katzung 13/e p754. 12/e p776; KDT 7/e
p344, 6/e p334; 19/e p2.
Esophageal irritation:
Minimized by taking the drug with full glass of water and remaining
upright for 30 minutes or by using the intravenous forms of these
compounds.
GI side effects:
Oral bisphosphonates (including alendronate, ibandronate &
risedronate) - Cause heartburn, esophageal irritation /esophagitis.
Also cause abdominal pain & diarrhea.
Symptoms often abate when patients take the medication after an
overnight fast, with tap or filtered water (not mineral water), and
remain upright.
46. Absorption of which of the following
drugs is increased after a fatty meal?
a) Amphotericin B

b) Griseofulvin

c) Ampicillin

d) Aspirin

Correct Answer - B
Ans: B. Griseofulvin
(Ref: Goodman Gilman I2/e p1585; Katzung 13/e p632, 12/e p855,.
KUT 7/e p790, 6/e p760)
The oral administration of a 0.5 g dose of griseofulvin produces peak
plasma concentrations of 1µg/mL in 4 hours.
Improved drug absorption along with a fatty meal.
Drug Absorption
Drug Absorption increased by food
Reduced/Delayed by Food
Isoniazida Atovaquone° Lovastatin
Ampicillin°
Loratidine Carbamazepine° Methylphenidate
Aspirin°
Naficillin Chlorthiazide° Metoprolol°
Atenolol°
Penicillin G or Cefuroxime° Nelfinavir
Azithromycin°
V° Clofazimine° Nitrofurantoin
Captopril°
Phenobarbital° Diazepam° Propranolol°
Cefaclor
Phenytoin° Erythromycin Propoxyphene
Cephalexin
Rifampin° Ganciclovir° Ritonavir°
Ciprofloxacin
Sucralfate° Hydrochlorothiazide° Saquinavir°
Didanosine
Tetracycline° Itraconazolea Spironolactone°
Indinavir°
Doxycycline° Lithium° *Hydralazine°
47. Steroids do not have a role in
management of which of these tumors?
a) Kaposi sarcoma

b) Chronic lymphoid leukemia

c) Hodgkin's lymphoma

d) Multiple myeloma Gram-stain, oropharynx

e) PAS, intestine

Correct Answer - A
Ans: A. Kaposi sarcoma
(Ref Harrison 19/e p1270, 716; Goodman Gilman I2/e p1755;
Katzung 13/e p954-955, 12/e p706: KDT 7/e p284-285, 6/e p285)
Steroids - Therapeutic uses in cancer:
Used as cytotoxic agents in treatment of acute leukemia in children
& malignant lymphoma in children & adults.
Component of curative regimens for Hodgkin's & non-Hodgkin's
lymphoma, multiple myeloma & CLL.
Glucocorticoids - Extremely helpful in controlling autoimmune
hemolytic anemia & thrombocytopenia associated with CLL.
Dexamethasone - Used in conjunction with radiotherapy - Reduces
edema related to tumors in critical areas such as superior
mediastinum, brain & spinal cords.
48. Which of these anticonvulsants causes
contraction of visual field?
a) Levetiracetam

b) Phenytoin

c) Vigabatrin

d) Ethosuximide

Correct Answer - C
Ans: C. Vigabatrin
Vigabatrin:
Causes irreversible diffuse atrophy of the retinal nerve fiber layer.
Most effect on the outer area (as opposed to the macular, or central
area) of the retina, leading to the contraction of the visual field.
Retinal toxicity attributed to taurine depletion.
49. Which one of the following is a gender-
specific side-effect of valproate?
a) Polycystic ovarian syndrome

b) Alopecia

c) Weight loss

d) Tremor

Correct Answer - A
Ans: A. Polycystic ovarian syndrome
Side-effects of Valproic Acid
MC side effects are transient
GI symptoms (anorexia,
nausea & vomiting)Effects on
the CNS: Sedation, ataxia Elevation of hepatic transaminases,
& Rash, alopecia Stimulation microvesicular steatosisAcute
of appetite & weight pancreatitisHyperammonemiaNeural
gain.Increase the chance of tube defects
polycystic ovary syndrome
(PCOS) in women with
epilepsy or bipolar disorders

50. Etanercept is a disease-modifying drug
used in management of rheumatoid
arthritis. What is its mechanism of action?
a) Inhibition of TNF alpha

b) COX-2 inhibition

c) IL-6 inhibition

d) Stabilization of mast cells

Correct Answer - A
Ans: A. Inhibition of TNF alpha
Gilman 12/e p182; KDT7/e p883, 6/ep205).
Etanercept:
Fusion protein produced by recombinant DNA.
It fuses the TNF receptor to the constant end of the IgG1 antibody.
It reduces the effect of naturally present TNF.
Hence a TNF inhibitor, functioning as a decoy receptor that binds to
TNF.
51. Which of the following drugs is not used
in detoxifica-tion of chronic alcoholics?
a) Flumazenil

b) Disulfiram

c) Acamprosate

d) Naltrexone

Correct Answer - A
Ans: A. Flumazenil
(Ref: Goodman Gilman 12/e p468; Kaaung 13/e p3 7 7, 394, 12/e
p381, 399; KDT 6/e p385; Harrison 19/e p2727, 8/c 1)2727)
Flumazenil:
GABAa receptor antagonist.
Used intravenously to treat benzodiazepine overdoses.
Help reverse anesthesia.
Not used in alcohol detoxification.
Rest given drugs have a role in alcohol detoxification.
52. Shoe polish like smell is seen in:
a) Mercaptans

b) Lacquer

c) Paraldehyde

d) Nitrobenzene

Correct Answer - D
Ans: D. Nitrobenzene
(Ref Reddy 34/e p468, 33/e p507; Industrial Hygiene and
Toxicology, General Principles/p174) Shoe polish like smell is seen
in Nitrobenzene.
Nitrobenzene:
An oily yellow to yellow-brown liquid that smells like bitter almonds
or shoe polish.
Dissolves only slightly in water and easily in other chemicals.
Man-made.
Most common exposure is at workplaces that use nitrobenzene to
produce dyes, drugs, pesticides or some types of rubber.
53. Which acid does not show coagulation
necrosis on contact?
a) HC1

b) H,SO4

c) HF

d) HNO3

Correct Answer - C
Ans: C. HF
(Ref Reddy 34/e p493, 33/e p530; Principles of Clinical Toxicology
3/e p220; Forensic Pathology 3/ep241, 110).
HF does not show coagulation necrosis on contact.
Hydrofluoric acid causes liquefaction necrosis.
54. Ashley rule is for determination of what:
a) Age

b) Sex

c) Height

d) Ethnicity

Correct Answer - B
Ans: B. Sex
(Ref Modern Medical Toxicology/p4)
Ashley's rule is used to know the sex of sternum.
It is also known as '149 rule'.
Differences between Male & Female sternum
Features Male sternum Female sternum
Ashley's ruleTotal length >149 mm Total length <149 mm
55. A man throws sulphuric acid on the face
of his wife after a fight following which
she comes to the emergency for
supportive management. All of these
statements are true about chemical burns
except:
a) Blisters are present

b) Ulcerated patches are present

c) Absence of singeing of hairs

d) Coagulation necrosis occurs at the site of burn

Correct Answer - A
Ans: A. Blisters are present
(Ref Parikh 6/e p4.168) Vesicles & blisters are usually absent in
chemical burns.
Chemical burn:
Occurs when living tissue is exposed to a corrosive substance such
as a strong acid or base.
Follows standard burn classification.
Cause extensive tissue damage.
Main types of irritant and/or corrosive products are:
Acids, bases, oxidizersreducing agents, solvents, and alkylants.
Chemical weapons, e.g. vesicants such as mustard gas and lewisite,
or urticants such as phosgene oxime.
56. A doctor who performed the autopsy on a
26-year-old married woman, committing
suicide found the cause of death to be
aluminium phosphide poisoning. She was
summoned in a court of law where he
willingly hides this information. This is
punishable under which section?
a) IPC 193

b) CrPC 175

c) CrPC 69

d) IPC 189

Correct Answer - A
Ans: A. IPC 193
(Ref: Reaco 34/e p I 3, 33/e p392; Textbooi, on the Indian Penal
Code by Krishna 1) Gaur 4/e p594; the-Indian-penal-code-
pdfd74214920)
Section 193 - In Indian Penal Code deals with punishment for giving
false evidence.
57. Ewing's postulates concerns with which
of the following:
a) Growth at the site following trauma

b) Growth after a neurological injury

c) Age related changes in the teeth

d) Old seminal stains

Correct Answer - A
Ans: a. Growth at the site following trauma
(Ref Parikh 6/e p4.89).
Ewing's Postulates:
Certain criteria, known as `Ewing's postulates', must be
satisfied before a relationship between trauma and new growth
is accepted:
The tumor must arise exactly at the site injured
Definite and substantial trauma must be proved
The tumor must be confirmed pathologically
The tissue at the site must have been healthy before the trauma
A reasonable interval-neither too long or too short—must elapse
between the time of the trauma and the appearance of the tumor
Though not one of the Ewing's original postulates, there should be
some good scientific reason for ascribing the tumor formation to the
injury and this is rarely possible.
58. Boiled lobster syndrome is seen in
poisoning of:
a) Boric acid

b) HNO,

c) H,SO4

d) Phenol

Correct Answer - A
Ans: A. Boric acid
(Ref: Principles of Clinical Toxicology 3/e p221).
Boiled lobster syndrome is seen in poisoning of Boric acid.
Features:
Major symptom is erythema, desquamation and exfoliation.
The skin of the patient looks like a 'boiled lobster'.
59.
An autopsy was performed on a case of
accidental death. It showed two linear
fractures on petrous part of temporal bone.
Which of the following rules gives the
sequence of fractures?
a) McNaughton's rule

b) Puppe's rule

c) Young's rule

d) Dunlop's rule

Correct Answer - B
Ans: B. Puppe's rule
(Ref. Reddy 34/e p213. 33/e p225, Forensic Pathology 3/e p241,
110)
Puppe's rule:
Used to determine that which fracture line has occurred before the
second one.
60. Which of these findings is not specific of
blast injury?
a) Abrasion

b) Bruise

c) Puncture laceration

d) Fracture

Correct Answer - D
Ans: D. Fracture
(Ref: Reddy 34/e 1)225-227, 33/e p239-240; Parikh 6/e p4.183;
Sabiston 19/e p6I2-613; Bailey 26/e p430 25/e p422-423)
Fracture is not specific of blast injury.
Marshall's triad:
Diagnostic of explosive injury.
Includes bruises, abrasions & puncture lacerations.
61. According to National Crime Bureau, most
common mode of suicide in India is:
a) Drowning

b) Poisoning

c) Fall from height

d) Hanging

Correct Answer - D
Ans: D. Hanging
According to National Crime Bureau, most common mode of suicide
in India is hanging.
Prominent means of committing suicides
Hanging (41.8%) Self-Immolation (6.9%)
Consuming Poison (26.0%) Drowning
62. Which of the following statement is false
statement about snake-bites?
a) Anti-venom is not effective in humpnosed pit viper bite

b) Cobra venom is neurotoxic

c) Atropine premedication should be used before administering


Neostigmine

d) Neostigmine has a role in krait bite.

Correct Answer - D
Ans: D. Neostigmine has a role in krait bite.
(Ref: Parikh 6/e p9.47: Harrison 19/e p2736; Snake Bite: Indian
Guidelines and Protocol p 425)
Role of neostigmine in snake-bite:
An anticholinesterase.
Particularly effective in postsynaptic neurotoxins (cobra).
Not useful against presynaptic neurotoxin (common krait & Russell's
viper).
Polyvalent Anti-snake venom - Ineffective against humpnosed pit
viper (Hypnale).
63. A 35-year-old male farmer presents with
multiple discharging cervical sinuses.
Which of these stains will be useful for the
diagnosis? Where does this organism
normally colonise in the body?
a) Grocott Methenamine silver, skin

b) PAS, intestine

c) AFB, mouth

d) Gram-stain, oropharynx

Correct Answer - C
Ans: C. AFB, mouth
(Ref Ananthanarayan 10/e p398, 8/e p391-391' I larrison 19/e
p1088)
In this farmer, who presented with multiple discharging cervical
sinuses, the most likely diagnosis is actinomycosis.
Actinomyces - Not acid-fast organism.
Actinomycosis:
Occurs most commonly at an oral, cervical, or facial site, usually as
a soft tissue swelling, abscess, or mass lesion that is often mistaken
fir a neoplasm.
The angle of the jaw is generally involved, but a diagnosis of
actinomycosis should be considered with any mass lesion or
relapsing infection in the head and neck.
64. A patient comes to your clinic with a
complaint of multiple episodes of loose
watery stool for 3 days. On probing, you
discover that these episodes start after he
had ingested shellfish at a local restaurant
3 days back and other people who had
food from that restaurant had similar
symptoms. What is the most common
cause of viral diarrhoea in adults?
a) Calicivirus

b) Rotavirus

c) Adenovirus

d) Norovirus

Correct Answer - D
Ans: D. Norovirus
Ref Harrison 19/e p1285, 1286; Jawetz 27/e p537
History of multiple episodes of loose watery stool for 3 days &
history of ingestion shellfish with similar symptoms in other patients
who consumed the shellfish is suggestive of acute viral
gastroenteritis.
Most likely organism responsible – Norovirus.
Noroviruses:
Most common infectious agents of mild gastroenteritis in the
community and affect all age groups, whereas sapoviruses primarily
cause gastroenteritis in children.
65. Antibody-dependent enhancement is
implicated in the immunopathogenesis of
which disease?
a) Influenza

b) Staphylococcal toxic shock syndrome

c) Waterhouse-Friderichsen syndrome

d) Dengue hemorrhagic fever

Correct Answer - D
Ans: D. Dengue hemorrhagic fever
Antiborly-dependent enhancement is implicated in the
immunopathogenesis of Dengue hemorrhagic fever.
66. Phage mediated change in C. diphtheria is
due to which of the following?
a) Conjugation

b) Transformation

c) Transduction

d) None of the above

Correct Answer - C
Ans: C. Transduction
(Ananthanarayan 10/e p59, 8/e p65-67; Jawetz 27/e p112).
Phage mediated change in C. diphtheria is due to Transduction.
In transduction, donor DNA is carried by a phage coat and is
transferred into the recipient by the mechanism used for phage
infection.
Methods of
Mechanism Nature of DNA transferred
transfer
Transfer of
Conjugation DNA form one Chromosomal or plasmid DNA°
bacterium to
Transfer of Any gene in generalized
Transduction DNA form one transduction, only selected genes in
bacterium to specialized transduction°
Transfer of
Transformation DNA from one Any gene°
bacterium to
67. A latex agglutination test for detection of
meningitis was approved. Calculate the
sensitivity and specificity of the test
based on the data given below:
Test PositiveTest Negative
Diseased 27 3
Non-diseased5 95

a) Sensitivity 90% Specificity 95%

b) Specificity 90% Sensitivity 95%

c) Sensitivity 80% Specificity 90%

d) Sensitivity 75% Specificity 95%

Correct Answer - A
Ans: A. Sensitivity 90% Specificity 95%
Sensitivity:
Introduced as a statistical index of diagnostic accuracy.
Defined as the ability of a test to identify correctly all those who have
the disease, that is 'true positive:"
Specificity:
Measures the proportion of negatives that are correctly identified as
such, e.g., the percentage of healthy people who are correctly
identified as not having the condition, that is true negative.
Sensitivity = Proportion of persons with the condition who test
positive: a /(a + c)`t = 27/30 = 90% .
Specificity = Proportion of persons without the condition who test
negative: d /(b + d)° = 95/100 = 95%.
68. A patient of diabetes and hypertension
comes to your clinic. As a doctor, you
explain to him the risks of various
complications. Which of these is the best
tool to demonstrate the complications?
a) Pie chart

b) Histogram

c) Scatter plot

d) Venn diagram

Correct Answer - D
Ans: D. Venn diagram
(Ref https://en.itikipedia.org/wiki/Venn_diagram)
A Venn diagram is an easy tool to make people understand
overlapping risk factors, complications and comorbidities to a patient
in a simple language.
69. Which of these is the best study to
evaluate effect and outcome?
a) Clinical trial

b) Cohort

c) Case control study

d) Cross sectional study

Correct Answer - A
Ans: A. Clinical trial
Among the given options, clinical trial is the best study to evaluate
effect and outcome.
Randomized control trial (Experimental study or Intervention
study):
Considered as the ideal design to evaluate the effectiveness and the
side-effects of new forms of intervention.
Cohort Study:
A study design where one or more samples (called cohorts) are
followed prospectively and subsequent status evaluations with
respect to a disease or outcome are conducted to determine which
initial participants exposure characteristics (risk factors) are
associated with it.
70. A researcher selected all possible
samples from a population and plotted
their means on a line graph. This
distribution is called as:
a) Sample distribution

b) Sampling distribution

c) Population distribution

d) Parametric distribution

Correct Answer - B
Ans: B. Sampling distribution
(Ref Research Methods For Communication Science James H.
Watt;
Sampling distribution / finite-sample distribution:
Probability distribution of a given statistic based on a random
sample.
They allow analytical considerations to be based on the sampling
distribution of a statistic, rather than on the joint probability
distribution of all the individual sample values.
71. Some medicine comes with a label of
'store at a cool place only'. At what
temperature should these medicines be
kept?
a) 8-15 °C

b) 2-8 °C

c) 0 °C

d) 25-28 °C

Correct Answer - A
Ans: A. 8-15 °C
(Ref: Park 22/e p100)
Some medicines come with label of 'store at a cool place only'.
These medicines should be kept at 8-15 °C.
Definitions of Storage Conditions of Drugs as per Ip 6 (Indian
Pharmacopoeia 1996)
Any temperature not exceeding 8°C and usually
between 2-8°C. A refrigerator is a cold place in which
Cold
the temperature is maintained thermostatically
between 28°C.
Any temperature between 8-25°C.
An article, for which storage in a cool place is directed
Cool
may alternately,be stored in a refrigerator unless
otherwise specified in the individual monograph.
Room
The temperature prevailing in a working area.
Temperature
Warm Any temperature between 30-40°C.
Excessive
Excessive
heat Any temperature above 40°C.
Light A light-resistant container protect the content from
resistant the effect of actinic light by virtue of the specific
containers properties of the material of which it is made.
A well-closed container protects the contents from
Well closed contamination by extraneous liquid & from loss of
container the article under normal condition of handling,
shipment, storage & distribution.
A tightly closed container protects the contents from
Well closed contamination by extraneous liquid & solids or vapor, from
loss or deterioration of the article from effervescence, deliquescent or
container evaporation under normal condition of handling, shipment, storage &
distribution.

72. The mean systolic blood pressure was
measured in a sample population of
elderly females and came out to be 125
mm Hg with a standard deviation of 10. 95
percent of people would have blood
pressure above:
a) 105 mm Hg

b) 110 mm Hg

c) 115 mm Hg

d) 140 mm Hg

Correct Answer - B
Ans: B. 110 mm Hg
(Ref: Park 24/e p 886)
73. A drug, which does not cure a disease but
decreases its symptoms and increases
survival, leads to?
a) Increased prevalence

b) Increased incidence

c) Decreased prevalence

d) Decreased incidence

Correct Answer - A
Ans: A. Increased prevalence
(Ref Park 24/e p66, 23/e p61, 22/e p5()
A drug, which does not cure a disease but decreases its symptoms
and increases survival, means its leads to increased duration of
disease and increased prevalence.
Prevalence = No. of total (old + new) cases of a disease in a
year/Total population 'x 100 Prevalence = Incidence x Mean duration
of disease
74. Which of these is not true about
randomization in a clinical trial?
a) Reduces confounding

b) Decreases selection bias

c) Ensures comparability of two groups

d) Increases external validity of the trial

Correct Answer - D
Ans: D. Increases external validity of the trial
(Ref Park 24/e p87, 23/e p82, 22/e p79)
External validity is the validity of generalized (causal) inferences in
scientific research, usually based on experiments as experimental
validity.
In other words, it is the extent to which the results of a study can be
generalized to other situations and to other people.
This cannot be ensured by randomization.
75. Which of these statements is true about
Rashtriya Swasthya Bima Yojana?
a) Cashless benefit on presenting smartcard and fingerprints

b) Valid for up to 4 family members

c) Can be used only in 1 district

d) Treatment only in government hospitals

Correct Answer - A
Ans: A. Cashless benefit on presenting smartcard and
fingerprints
Rashtriya Swasthya Bima Yojana (RSBY):
Government- run health insurance scheme for the Indian poor.
It provides fir cashless insurance for hospitalization in public as well
as private hospitals.
Provide health insurance coverage for Below Poverty Line (BPL)
families.
Objective is to provide protection to BPL households from financial
liabilities arising out of health shocks that involve hospitalization.
76. A recently delivered woman with a 15
days old child suffering from cough,
sneezing and fever needs help. She has
no money for transportation to nearby
hospital. Which of the national
programme can help this woman?
a) JSSK

b) Indira Gandhi YojanaF-IMNCI

c) F-IMNCI

d) Home-based Care

Correct Answer - A
Ans: A. JSSK
(Ref Park 24/e p476, 23/e p456, 22/e p420;
http://nrhm.gov.in/janani-shishu-suraksha-karyakram.html)
Janani Shishu Suraksha Karyakaram (JSSK), the national
programme can help this woman.
Janani-Shishu Suraksha Karyakram (JSSK)
The initiative entitles all pregnant women delivering in public health
institutions to absolutely free and no expense to delivery, including
cesarean section.
Includes free drugs and consumables, free diet up to 3 days during
normal delivery and up to 7 days for cesarean section, free
diagnostics, and free blood wherever required.
Provides for free transport from home to institution, between facilities
in case of referral and drop back home.
Similar entitlements for all sick newborns & infants accessing public
health institutions for treatment till 30 days after birth.
Aims to eliminate out of pocket expenses incurred by the pregnant
women and sick new borns while accessing services at Government
health facilities.
77. According to the new WHO 2013 malaria
treatment guidelines, which of the
following statements is true?
a) No ACT in falciparum malaria

b) Presumptive treatment with chloroquine should be given

c) Primaquine is contraindicated in infants and pregnant women

d) Primaquine is to be given for 7 days in falciparum malaria

Correct Answer - C
Ans: C. Primaquine is contraindicated in infants and pregnant
women
Primaquine:
In G6PD deficiency persons.
Contraindicated during pregnancy & in lactation.
Unless the infant being breast-fed has a documented normal G6PD
level.
78. The trivalent influenza vaccine contains
all of the following strains except:
a) H IN I

b) H3N2

c) H2N 1

d) Influenza B

Correct Answer - C
Ans: C. H2N 1
Each seasonal influenza vaccine contains antigens representing
three (trivalent vaccine) or four (quadrivalent vaccine) influenza virus
strains: one influenza type A subtype H1 N1 virus strain, one
influenza type A subtype H3N2 virus strain, and either one or two
influenza type B virus strains.
79. Most cost-effective method of infection
control is:
a) Repeated disinfectant use

b) Alcohol based rubbing

c) Prophylactic antibiotic therapy

d) Hand washing

Correct Answer - D
Ans: D. Hand washing
(Ref Park 24/e p378, 22/e p3)
In developing countries, handwashing with soap is recognized as a
cost-effective, essential tool for achieving good health, and even
good nutrition.
80. Which is the most common type of
congenital ossicular dysfunction?
a) Isolated stapes defect

b) Stapes defect with fixation of footplate and lenticular process


involvement.

c) Defective lenticular process of incus

d) None of the above.

Correct Answer - B
Ans: B. Stapes defect with fixation of footplate and lenticular process
involvement.
(Ref Nelson 20/e p3071)
Most common type of congenital ossicular dysfunction is stapes
defect with fixation of footplate and lenticular process involvement.
81. Auditory neurotherapy is an effective
modality of treatment for which of the
following abnormalities of hearing?
a) CSOM

b) Meniere's disease

c) Malignant otitis externa

d) Otosclerosis

Correct Answer - B
Ans: B. Meniere's disease
(Ref: Dhingra 7/e p115, 6/100-104, 5/e pg 112-113: Logan-Turner
10th/335, Scott-Brown 7/e p3570)
Among the given options, only Meniere's disease involves the
vestibular system of inner ear.
Hence, a neurotherapy (direct nerve stimulation) is going to be
useful only in Meniere's disease.
82. Which of the following tests is
recommended for neonatal screening of
hearing?
a) Automated auditory brainstem response

b) Spontaneous OAE

c) Evoked OAE

d) Distorted product OAE

Correct Answer - C
Ans: C. Evoked OAE
(Ref: Diungra 7/e p29-30. 5/e p116)
Evoked otoacoustic emission (OAE) is recommended for neonatal
screening of hearing.
Types of Otoacoustic Emissions (OAE)
SOAE are seen in 25-
Sounds emitted
80% of neonates with
Spontaneous without an acoustic
normal hearing &
OAE (SOAEs) stimulus
absence of SOAEs is not
(i.e, spontaneously).
necessarily abnormal
TOAEs commonly are
Sounds emitted in
used to screen infant
response to an
Transient hearing, to validate
acoustic stimuli of
evoked OAE behavioural or
very short duration;
(TEOAEs) electrophysiologic
usually clicks but
auditory thresholds & to
can be tone-bursts
assess cochlear function
Sounds emitted in Particularly useful for
Distortion response to early detection of
product OAE 2 simultaneous cochlear damage as they
(DPOAEs) tones of are for ototoxicity &
different frequencies noise-induced damage
Sounds emitted in SPOAEs are responses
Sustained- frequency
response to recorded to a continuous
OAE (SFOASEs)
a continuous tone tone.
83. A patient presents to your clinic for
evaluation of defective hearing. Rinne's
test shows air conduction greater than the
bone conduction on both sides with
Weber test lateralized to right ear. What is
the next logical step?
a) Normal test

b) Schwabach's test

c) Repeat Rinne's test on right side

d) Wax removal

Correct Answer - B
Ans: B. Schwabach's test
(Ref Dhingra 7/e p23, 5/e p25-27)
Lateralization of Weber's test to right implies either right-sided
conductive deafness or left sided sensorineural deafness.
Now in conductive deafness bone conduction is better than air
conduction in Rinnie's test, hence the patient probably has
sensorineural deafness involving the left ear, as per the findings of
Rinnie's and Weber tests.
In such a case, Schwabach's test should be performed to see the
absolute bone.
84. Keyhole-shaped visual field defect is seen
in lesion involving which of the following
regions?
a) Optic disk

b) Optic chiasma

c) Lateral geniculate body

d) Occipital lobe

Correct Answer - C
Ans: C. Lateral geniculate body
(Ref Walsh and Hoyt; Clinical Neuro-Ophthalmology 6/e p122)
Key-hole shaped visual field defects are typically seen in the lesions
involving lateral geniculate body but keyhole shaped defect (not
visual field defect) is seen in the coloboma of Iris.
85. Which of the following is the most
common funga. infection of the eye seen
in an HIV positive patient?
a) Aspergillus

b) Candida

c) Toxoplasma

d) Rhinosporidium

Correct Answer - B
Ans: B. Candida
(Ref.: Ryan's Retina 5/e p733)
Candidemia:
Most common fungal infection seen in patients with HIV
Candida albicans - Important nosocomial pathogen - Most common
Candida species.
Most commonly cause of keratitis, conjunctivitis and endogenous
fungal endophthalmitis.
86. A 29-year-old male athlete suddenly
collapsed and died during a football
game. At autopsy the following finding
was seen on gross examination. He had a
history of two similar deaths in the family
previously. What is the most likely cause
of death?
a) Hypertrophic cardiomyopathy

b) RHD

c) Coronary heart disease

d) Dilated cardiomyopathy

Correct Answer - A
Ans: A. Hypertrophic cardiomyopathy
(Ref. Horrison 19/e p1568, 18/e p1967).
Suggestive of Hypertrophic cardiomyopathy.
Hypertrophic cardiomyopathy:
Macroscopically. hypertrophy is typically manifest as nonuniform
ventricular thickening.
The interventricular septum is the typical location of maximal
hypertrophy, although other patterns of hypertrophic remodeling
include concentric and mid-ventricular.
87. All of these are true about
microalbuminuria
a) Urine protein levels range from 20 mg/d to 200 mg/d

b) It is an independent risk factor for cardiovascular morbidity in


diabetic patients

c) It is the earliest marker of diabetic nephropathy

d) It is not detected by routine dipstick method

Correct Answer - A
Ans: A. Urine protein levels range from 20 mg/d to 200 mg/d
Refs Harrison 19/e p2425, 1582, 1813 18/e p2982, 29(S
Microalbuminuria:
Defined as 30-299 mg/d in a 24-h collection or 30-299 mcg/mg
creatinine in a spot collection.
The American Diabetes Association (ADA) recently suggested that
the terms previously used to refer to increased urinary protein
(microalbuminuria as defined as 30-299 mg/d in a 24-h collection or
30-299µg/mg creatinine in a spot collection or macroalbuminuria as
defined as >300 mg/24 h) be replaced by the phrases "persistent
albuminuria (30-299 mg/24 lz)" and "persistent albuminuria (>300
mg/24 h)" to better reflect the continuous nature of albumin excretion
in the urine as risk factor for nephropathy and cardiovascular
disease (CVD).
88. Which of the following is a new drug
available to treat multi drug resistant
tuberculosis?
a) Bedaquiline

b) Rifampicin

c) Linezol id

d) Cefepime

Correct Answer - A
Ans: A. Bedaquiline
(Ref. Harrison I9/e p11I5, 205e-7)
Two novel drugs belonging to two new antibiotic classes—the
diarylquinoline bedaquiline and the nitroimidazole delamanid—have
recently been approved for use in severe cases of MDR-TB by
stringent regulatory authorities.
89. According to WHO guidelines, latent TB
should be ruled out in all the following
situations except:
a) Before treatment with TN F-alpha inhibitors

b) Chronic alcoholics

c) Silicosis

d) Hemodialysis

Correct Answer - B
Ans: B. Chronic alcoholics
According to WHO guidelines, latent TB should be ruled out before
treatment with TNF-alpha inhibitors, in patients of silicosis and
patients undergoing hemodialysis among the provided options.
Specific High-risk Populations
Silicosis°
HIV infection° Being in prison
Recent contact with an infectious patient Being an immigrant
Initiation of an anti-tumor necrosis factor from high
(TNF) treatment° TB burden countries
Receiving dialysis° Being a homeless
Receiving an organ or person
hematologic transplantation Being an illicit drug
user
90. A65-year-old male adult presents with
chronic sinusitis, nasopharyngeal ulcers,
cavitatory lung nodules and renal failure.
What will be the appropriate next
diagnostic step?
a) Lung biopsy

b) Sputum AFB and PCR for TB

c) ANCA and evaluation for vasculitis

d) ESR

Correct Answer - C
Ans: C. ANCA and evaluation for vasculitis
(Ref Harrison 19/e p2182-2184, 18/e p2786, 2789)
Suggestive of Wegener's Granulomatosis.
Wegener's Granulomatosis:
c-ANCA positive small vessel vasculitis.
levels are required for diagnosis.
Tests:
Specificity of a positive antiproteinase-3 ANCA for granulomatosis
with polyangiitis (Wegener's) is very high, especially if active
glomerulonephritis is present.
However, the presence of ANCA should be adjunctive and, with rare
exceptons, should not substitute for a tissue diagnosis.
91. Serology profile done for a patient is
mentioned below. What is the likely cause
of the abnormal findings:
HbsAg-Non-reactive
HBV DNA-Undetectable
HbeAg-Non-reactive
IgG Anti-HbC-Reactive
a) Chronic hepatitis inactive state

b) Chronic hepatitis recovery state

c) Pre-core mutant infection

d) Window period

Correct Answer - B
Ans: B. Chronic hepatitis recovery state
From the given serology profile and table given below it is clear that
the patient has had hepatitis B infection in the remote past or is a
low-level Hepatitis B carrier.
Commonly Encountered Serologic Patterns of Hepatitis B
Infection
Anti- Anti- Anti-
HBsAg HBeAg Interpretation
HBs HBc HBe
+ – IgM + – Acute hepatitis B, high infectivity
Chronic hepatitis B, high
+ – IgG + –
infectivity°
Late acute or chronic hepatitis B,
low infectivity°HBeAg-negative
+ – IgG – + ('precoremutant') hepatitis B
(chronic or rarely acute)

HBsAg of one subtype and


heterotypic anti-HBs
+ + + +1– +1– (common)Process of
seroconversion from HBsAg to
anti-HBs (rare)
Acute hepatitis B°Anti-HBc
– – IgM +/– +/–
`window'0
Low-level hepatitis B
– – IgG – +/– carrier°Hepatitis B in remote
past°
– + IgG – +/– Recovery from hepatitis B°
Immunization with HBsAg (after
vaccination)
– + – – –
Hepatitis B in the remote past (?)
False-positive

92. A 23-year-old male patient presented with
a history of back pain, which is more in
the morning and relieved by bathing in
warm water. What is the likely additional
finding present in this patient?
a) Marrow fibrosis

b) Distal phalangeal joint involvement

c) Pleural nodules

d) Decreased chest wall expansion

Correct Answer - D
Ans: D. Decreased chest wall expansion
(Ref -Harrison I9/e p2170, 18/e p2775)
Suggestive of spondylo arthropathy, like ankylosing spondylitis.
Ankylosing Spondylitis:
Leads to extra parenchymal restrictive lung disease, associated with
decreased chest wall expansion.
Initial symptom is usually dull pain, insidious in onset, felt deep in the
lower lumbar or gluteal region, accompanied by low-back morning
stiffness of up to a few hours' duration that improves with activity and
returns following inactivity.
93. Which of these is a new oral drug used in
treatment of chronic Hepatitis C?
a) Interferon alpha

b) Ledipasvir

c) Oseltamivir

d) Lamivudine

Correct Answer - B
Ans: D. Lamivudine
(Ref: Harrison 19/e p2049, 18/e p2556)
Ledipasvir:
Inhibitor of hepatitis C virus NS5A protein.
Daclatasvir, Ledipasvir is one of the newer drugs for hepatitis C.
Most commonly used in combination with sofosbuvir for treatment in
chronic hepatitis C genotype 1 patients.
An inhibitor of the hepatitis C virus NS5A protein.
Sofosbuvir:
Metabolized to the active uridine analog triphosphate.
Acts as a RNA chain terminator when incorporated into the RNA via
the NS5B polymerase.
94. In a patient of jaundice, absence of
urobilinogen in urine indicates?
a) Obstructive jaundice

b) Hemolysis

c) Liver failure

d) Hepatitis

Correct Answer - A
Ans: A. Obstructive jaundice
(Ref Harrison 19/e p280, 18/e p325)
Absence of urinary urobilinogen is usually suggestive of obstructive
jaundice.
Prehepatic Hepatic Post-
Function test
jaundice jaundice hepatic (Obstructive)jaundice
Normal/
Total bilirubin Increased° Increased°
increased
Conjugated
Normal Increased° Increased°
bilirubin
Unconjugated Normal/
Increased° Normal
bilirubin increased
Normal/
Urobilinogen Decreased° Decreased/ negative°
increased
Dark
(urobilinogen
Urine color Normal Dark (conjugated bilirubin)°
+ conjugated
bilirubin)°
Stool color Normal Normal/pale° Pale°
Alkaline
Normal Increased° Increased°
Normal Increased° Increased°
phosphatase levels
Alanine transferase
&
Normal Increased° Increased°
aspartate transferase
levels
Conjugated bilirubin
Absent° Present° Present°
in urine
Absent°
Splenomegaly Present° Present°
95. Most specific sign of metabolic
encephalopathy is:
a) Asterixis

b) Abulia

c) Akinetic mutism

d) Apraxia

Correct Answer - A
Ans: A. Asterixis
(Ref Harrison 19/e p1782. 2066, 1774. 18/e p199. 2259)
The presence of asterixis on the motor examination is nonspecific
but usually indicates a metabolic or toxic etiology of the delirium.
In a drowsy and confused patient, bilateral asterixis is a certain sign
of metabolic encephalopathy or drug intoxication.
Hepatic encephalopathy is suggested by asterixis and can occur in
chronic liver failure or acute fulminant hepatic failure.
96. A patient presented with a steering wheel
injury to the right side of chest with
breathlessness and shock. How will you
differentiate tension pneumothorax and
cardiac tamponade?
a) Pulse pressure

b) JVP

c) Breath sound

d) Heart sounds

Correct Answer - C
Ans: C. Breath sound
(Ref Harrison 19/e p1573, 1719, 18/e p1972)
Raised JVP, pulse pressure and muffled heart sounds along with
pulsus paradoxus are seen in both cardiac tamponade and tension
pneumothorax.
These are differentiated by auscultation for breath sounds, which are
normal in cardiac tamponade but absent in tension pneumothorax
patients.
Pericardial Tamponade:
The three principal features of tamponade (Beck's triad) are
hypotension, soft or absent heart sounds, and jugular venous
distention with a prominent x-descent but an absent y-descent.
97. All of the following are features of
glucocorticoid deficiency except:
a) Fever

b) Hyperkalemia

c) Postural hypotension

d) Weight loss

Correct Answer - B
Ans: B. Hyperkalemia
(Ref: Harrison 19/c p2325, 18/c p2957)
Hyperkalemia is seen in mineralocorticoid deficiency but not in
isolated glucocorticoid deficiency.
Fever is due to release of inflammatory mediators, which is
suppressed by steroids, is seen in glucocorticoid deficiency.
98. Naming and fluency is impaired in:
a) Broca's aphasia

b) Wernicke's aphasia

c) Anomie aphasia

d) Transcortical sensory aphasia

Correct Answer - A
Ans: A. Broca's aphasia
(Ref Harrison 19/e p279, I8/e p203. 3286)
Repetition of
Comprehension Naming Fluency
spoken language
Preserved or
Wernicke's Impaired° Impaired° Impaired°
increase&
Preserved
Broca's (except grammar) Impaired° Impaired° Decrease&
°
Global Impaired° Impaired° Impaired° Decreased
Conduction Preserved Impaired° Impaired° Preserved
Nonfluent
Preserved Preserved Impaired Impaired
(motor) transcortical
Fluent
Impaired Preserved Impaired Preserved
(sensory) transcortical
No
Isolation Impaired Echolalia° Impaired purposeful
speech

Anomic Preserved° Preserved Impaired word? finding


Impaired only
Pure word deafness for spoken Impaired PreservedPreserved
language
Impaired only for
Pure alexia Preserved PreservedPreserved
reading
99. A 9-year-old boy presented with difficulty
in climbing stairs and combing. On
examination, bilateral calves are swollen
and the child uses his feet to stand up on
his legs. What is the next diagnostic step?
a) Creatine kinase levels

b) EMG

c) Nerve conduction velocity

d) RA factor

Correct Answer - A
Ans: A. Creatine kinase levels
(Ref Harrison 19/e p462e-6, 18/e p3490-3491)
Suggestive of Duchenne muscular dystrophy (Pseudohypertrophic
Muscular Dystrophy).
Another evidence is that child uses his feet to stand up on his legs
(Gower's sign).
Next diagnostic step is creatine kinase levels.
100. All are features of benign intracranial
hypertension except:
a) Proptosis

b) Normal size ventricles

c) Headache

d) Papilledema

Correct Answer - A
Ans: A. Proptosis
Proptosis is not seen in benign intracranial hypertension.
Characteristic features of Pseudotumor Cerebri:
Elevated intracranial pressure (intracranial hypertension) with
normal or small sized ventricular system.
No focal neurological sign.
Papilledema (enlarged blind spot in visual fluid).
Normal CSF finding
Normal CT scan
MRI & isotope brain scans
Excessive slow-wave activity on ECO2.
101. Peau d'orange in carcinoma breast is
due to:
a) Obstruction of sub-dermal lymphatics

b) Infiltration of Cooper's ligament

c) Hematogenous dissemination

d) Hematogenous dissemination

Correct Answer - A
Ans: A. Obstruction of sub-dermal lymphatics
Peau-d-orange is due to cutaneous lymphatic edema.
Caused by obstruction of subdermal lymphatics.
As the disease advances locally there may be skin involvement with
peau d'orange or frank ulceration and fixation to the chest wall.
This is described as cancer-en-cuirasse when the disease
progresses around the chest wall.
102. A patient underwent thyroidectomy for
hyperthyroidism. Two days later he was
presented with features of thyroid storm.
What is the most likely cause?
a) Poor antibiotic coverage

b) Rough handling during surgery

c) Removal of parathyroid

d) Inadequate preoperative preparation

Correct Answer - D
Answer D
Thyrotoxic crisis (storm)- This is an acute exacerbation of
hyperthyroidism.
It occurs if a thyrotoxic patient has been inadequately prepared for
thyroidectomy and is now extremely rare.
Features: They present in 12-24 hours after surgery; with severe
dehydration, circulatory collapse, hypotension, hyperpyrexia,
tachypnoea, hyperventilation, palpitation, restlessness, tremor,
delirium, diarrhea, vomiting, and cardiac failure; later coma.
Treatment: Injection hydrocortisone, oral antithyroid drugs, tepid
sponging of the whole body, beta-blocker injection, oral iodides, a
large amount of IV fluids for rehydration, digitoxin, cardiac monitor,
often ventilator support, with close observation.
103. A 60-year-old chronic smoker presented
with progres-sive jaundice, pruritus and
clay colored stools for 2 months. History
of waxing and waning of jaundice was
present. A CT scan revealed dilated main
pancre-atic duct and common bile duct.
What is the likely di-agnosis?
a) Carcinoma head of pancreas

b) Periampullary carcinoma

c) Chronic pancreatitis

d) H lar cholangiocarcinoma

Correct Answer - B
Ans: B. Periampullary carcinoma
(Ref Sabiston 201e p1544, 19/e p1535-1544; Schwartz 10/e p1408,
9/e p1220-1225 Bailey 27/e p1234, 26/e p1138, Blumgart 5/e p919-
925; Shackelford 7/e p1190-1196)
Suggestive diagnosis is periampullary carcinoma.
Since history of waxing and waning episode is seen.
The waxing and waning nature of jaundice is due to sloughing of
ampullaty cancer, resulting in transient resolution of the jaundice.
104. What is false about Meckel's
diverticulitis?
a) Present in 3% of the population.

b) Presents with periumbilical pain.

c) Remnant of proximal part of vitellointestinal duct.

d) Lies on the anti-mesenteric border.

Correct Answer - A
Ans: A. Present in 3% of the population.
(Ref .Sabo Ion 2(ke p1284, 19/e 1)1268-12;0; 3cliwarlz 10/e p1164,
9/e p1002- I OW. 1435: Bailey 27/e p1252, 26/e p1160-1170;
Shackelford 7/e p695-698)
Meckel's diverticulum:
Present in 2% of the population and not 3%.
Persistent remnant of the vitellointestinal duct.
Found on the antimesenteric side of the ileum, commonly at 60 cm
from the ileocaecal valve and is classically 5 cm long.
Contains all three coats of the bowel wall and has its own blood
supply.
105. A child presented with a swelling in the
right groin region. When the swelling
was reduced, a gurgling sound was
heard. Which of the following is an
incorrect statement?
a) The sac contains omentum only

b) The hernia lies above and medial to pubic tubercle

c) Patent processus vaginalis

d) This type of hernia is most common in children

Correct Answer - A
Ans: A. The sac contains omentum only
(Ref Sabiston 20/e p1097, 19/e p1114-1115; Schwartz 10/e p1503,
9/e p1307, 1316; Bailey 27/e p1031, 26/e p954-959; Schackelford
7/e p.565-568)
Gurgling sound on reduction in case of hernia indicates that the
content is bowel, not omentum.
106. During laparoscopic inguinal hernia
repair a tacker was accidently placed
below and lateral to the ilio-pubic tract.
Postoperatively the patient complained
of pain and soreness in the thigh. This is
due to the involvement of:
a) Lateral cutaneous nerve of thigh

b) llioinguinal nerve

c) Genital branch of genitofemoral nerve

d) Obturator nerve

Correct Answer - A
Ans: A. Lateral cutaneous nerve of thigh
(Ref Sabiston 19/e pink Schwartz 10/e p-1515, 9/e p1313-1315;
Sehackelford 7/e p562-565)
Neuropathic groin pain:
Caused by damage to a nerve in the groin region and may he due to
partial or complete division, stretching, contusion, crushing, suturing,
or electrocautery.
Nerves involved:
Ilioinguinal nerve, iliohypogastric nerve, both the genital and femoral
branches of the genitofemoral nerve, and the lateral femoral-1
cutaneous nerve of the thigh.
1st two more prone to injury during an open herniorrhaphy.
Latter (i.e. Lateral cutaneous nerve of thigh) are more likely
damaged during laparoscopy.
The genital and femoral branches of the genitofemoral nerve and the
lateral cutaneous nerve of the thigh are most at risk when the
surgeon staples below the iliopubic tract when lateral to the internal
.spermatic vessels.
A burning, tingling pain along the lateral aspect of the thigh in the
distribution of the lateral femoral cutaneous nerve is known as
meralgia paresthetica
Due to entrapment of that nerve.
Affected skin area:
Hyperaesthetic and/or pruritic.
Complain of tactile hallucination of a sensation of small insects
creeping under the skin (formication).
107. Which of these is the most reliable
method for monitoring fluid
resuscitation?
a) Urine output

b) CVP

c) Pulse rate

d) Blood pressure

Correct Answer - A
Ans: A. Urine output
Goal of treatment:
Restore cellular and organ perfusion.
Hence, monitoring of organ perfusion should guide the management
of shock.
The best measures of organ perfusion and the best monitor of the
adequacy of shock therapy remain the urine output.
108. What are the minimum and maximum
possible values of Glasgow Coma
Score?
a) Minimum = 3, Maximum = 15

b) Minimum = 0, Maximum = 13

c) Minimum = 0, Maximum = 15

d) Minimum = 3, Maximum = 18

Correct Answer - A
Ans: A. Minimum = 3, Maximum = 15
In Glasgow Coma Scale (GCS), the maximum score is 15 and the
minimum score is 3.
109. A patient had a massive bleeding during
surgery. Which sized cannula should be
used?
a) 16 Gauge

b) 20 Gauge

c) 22 Gauge

d) 24 Gauge

Correct Answer - A
Ans: A. 16 Gauge
(Ref Bailey 25/e p291)
In a patient with massive bleeding, there is always a risk of patient
going into shock and widest bore cannula available showed be used
for cannulation.
The ACLS guidelines recommend securing intravenous access with
two large-bore cannulae (14–I6G) in patient needing resuscitation.
Color External
Gauge Length Flow Rate
code Diameter
14G Orange°2.1 mm 45 mm 240 ml/min
16GGrey(' 1.8 mm 45 mm 180 ml/min
18GGreen° 1.3 mm 32/45 mm 90 ml/min
20GPink° 1.1 mm 32 mm 60 ml/min
22GBlue° 0.9 mm 25 mm 36 ml/min
24G Yellow°0.7 mm 19 mm 20 ml/min
26GViolee 0.6 mm 19 mm 13 ml/min
110. A 2-year-old child with fever and barking
cough for last 2 days presented to the
pediatric emergency at 2.30 am. On
examination, respiratory rate is 36/ min,
temperature of 39 °C and stridor heard
only on crying. No other abnormality is
found. What is the next best step in
management?
a) High-dose dexamethasone

b) Racemic epinephrine nebulization

c) Reassurance

d) Intravenous antibiotics

Correct Answer - A
Ans: A. High-dose dexamethasone
(Ref Ghai 8/e p376, 398: Nelson 20/e p2032-2034)
Diagnostic of laryngotracheobronchitis or croup of mild severity.
Hence, high-dose dexamethasone will be the treatment of choice.
Treatment:
Cornerstone of Treatment: Glucocorticoids & nebulized
epinephrine°
Glucocorticoids:
Useful in mild, moderate & severe croup.
Dexamethasone is most effective corticosteroid.
Nebulized epinephrine:
Useful in moderate to severe distress.
By adrenergic stimulation causes,
Constriction of precapillary arterioles.
Decreases capillary hydrostatic pressure leading to fluid resorption
from interstitium.
Improvement in laryngeal mucosal edema.
Antibiotics are not indicated
Heliox (mixture of oxygen & helium) has low viscosity & low specific
gravity.
Allows for greater laminar airflow through respiratory tract.
Considered in treatment of children with severe croup.
111. A 1-year old infant presents with 10-12
episodes of watery stools per day for the
last 9 days. Along with zinc
supplementation, what else should be
prescribed to the child?
a) ORS with antibiotics

b) ORS only

c) ORS with low-lactose diet

d) ORS with low-lactose diet and probiotics

Correct Answer - B
Ans: B. ORS only
The infant is suffering from acute diarrhea and treatment includes
oral rehydration therapy, zinc supplementation and continued
breastfeeding.
Low lactose diet is required in management of persistent diarrhea.
Antibiotics are required in management of dysentery, i.e. blood in
stools.
Routine use to probiotics in acute diarrhea is not recommended.
112. Milk is deficient in:
a) Iron and vitamin C

b) Iron and vitamin A

c) Phosphorus and vitamin A

d) Saturated fats

Correct Answer - A
Ans: A. Iron and vitamin C
Human breast milk has enough of all nutrients except Vitamin D and
Vitamin K. It is also slightly deficient in Vitamin C and iron.
Human vs Cow milk – nutritional value comparison:
Human milk is richer in carbohydrate (lactose), iron & water content.
Cow's milk is richer in fat, protein, calcium & energy content.
Human milk proteins:
More cystine & taurine; less methionine; better digested than cow's
milk proteins.
Human milk fats:
Higher levels of PUFAs, esp., linoleic acid & linoleic acid; better
digested and absorbed; low calcium content but better absorbed
than cow's milk.
Human milk is richer in Vitamin A & C; richer in copper, cobalt &
selenium; richer in iron & higher bioavailability; high
calcium/phosphorus ratio.
Human milk has lesser sodium.
113. All of these are criteria for severe acute
malnutrition in a 6-month-old child
except:
a) Mid-upper arm circumference

b) Symmetrical edema

c) Weight for height

d) Height for age

Correct Answer - D
Ans: D. Height for age
Height for age is not a criterion for severe acute malnutrition in a 6-
month old child.
Severe acute malnutrition (SAM):
]Among children 6-59 months of age is defined by World Health
Organization (WHO) and UNICEF as any of the following:
Weight-for-height below –3 standard deviation (SD or Z scores) of
the median WI-10 growth reference
Visible severe wastinge
Presence of bipedal edema
Mid-upper arm circumference below 11.5 cm
This classification is used to identify children at high risk of death.
114. How is under-nutrition defined?
a) Weight for age < –2 SD

b) Weight for height < –2 SD

c) Weight for age < –3 SD

d) Weight for height < –3 SD

Correct Answer - B
Ans: B. Weight for height < –2 SD
Under nutrition is defined in terms of Weight for Height, i.e.
WHO Classification of Malnutrition
Moderate
S
malnutrition
Symmetrical Yes (edematous
No
edema malnutrition)
SD score from –2 to SD score < –3 (severe
Weight-for-height
–3 wasting)
SD score from –2 to SD score < –3 (severe
Height-for-age
–3 stunting)

115. A 3.5 kg male infant born at term after an
uncomplicated pregnancy and delivery
develops respiratory distress shortly
after birth and requires mechanical
ventilation. The chest radiograph reveals
a normal cardiothymic silhouette but a
diffuse ground glass appearance to the
lung fields. Surfactant replacement fails
to improve gas exchange. Over the first
week life, the hypoxemia worsens.
Results of routing culture and
echocardiographic findings are negative.
A term female sibling died at 1 month of
age with respiratory distress. Which of
the following is the most likely
diagnosis?
a) Neonatal pulmonary alveolar proteinosis

b) Meconium aspiration

c) Total anomalous pulmonary venous return

d) Disseminated herpes simplex infection

Correct Answer - A
Ans: A. Neonatal pulmonary alveolar proteinosis
(Ref. Nelson 20/e p852, 2119)
Suggestive of neonatal pulmonary alveolar proteinosis.
Pulmonary alveolar proteinosis:
Disorder characterized by intra-alveolar accumulation of pulmonary
surfactant.
Two clinically distinct forms of pulmonary alveolar proteinosis
are seen:
Fatal form: Presenting shortly after birth (congenital PAP)
Gradually progressive form: Presenting in older infants & children.
Clinical manifestation:
Immediately apparent in the newborn period & rapidly leads to
respiratory failure.
Clinically and radiographically indistinguishable from more common
disorders of the newborn that lead to respiratory failure including
pneumonia, generalized bacterial infection, respiratory distress
syndrome and total anomalous pulmonary venous return with
obstruction.
116. Meralgia paresthetica is due to
involvement of:
a) Lateral cutaneous nerve of thigh

b) Genitofemoral nerve

c) llioinguinal nerve

d) Saphenous nerve

Correct Answer - A
Ans: A. Lateral cutaneous nerve of thigh
(Ref Apley:s 9/e p292)
The lateral cutaneous nerve can be compressed as it runs through
the inguinal ligament just medial to the anterior superior iliac spine.
The patient complains of numbness, tingling or burning discomfort
over the anterolateral aspect of the thigh (meralgia paraesthetica).
Testing for sensibility to pinprick will reveal a patch of numbness
over the upper outer thigh.
If the symptoms are troublesome - Nerve can be released.
117. All the following can lead to damage of
the axillary nerve except:
a) Fracture of surgical neck of humerus

b) Intramuscular injection

c) Improper use of crutches

d) Shoulder dislocation

Correct Answer - C
Ans: C. Improper use of crutches
(Ref Apley’s• 9/c p282)
Improper use of crutches causes radial nerve palsy mostly rather
than axillaty nerve injury.
Radial Nerve Injury:
Very high lesions may be caused by trauma or operations around
the shoulder.
Due to chronic compression in the axilla.
Seen in drink & drug addicts who fall into a stupor with the arm
dangling over the back of a chair - Saturday night palsy.
In thin elderly patients using crutches - Crutch palsy.
Weakness of wrist & hand.
Paralysed triceps.
Absent triceps reflex.
118. Which of the following statements is true
about Swyer svndrome?
a) Can be fertile with surrogacy

b) Can be fertile with ovum donation

c) Presents with primary fertility

d) Gonadectomy is indicated for all patients

Correct Answer - A
Ans: A. Can be fertile with surrogacy
(Ref Shaw's 16/e p145)
Swyer syndrome:
Patients can be fertile with surrogacy, with ovum donation.
Gonadectomy indicated for all patients due to malignancy risk.
119. Which of these is seen in Asherman
syndrome?
a) Oligomenorrhea

b) Hypomenorrhea

c) Metromenorrhagia

d) Polymenorrhea

Correct Answer - B
Ans: B. Hypomenorrhea
(Ref: Shaw's 16/e p250; Novaks 13/e p351)
Hypomenorrhea:
Seen in Asherman syndrome
Asherman's syndrome:
More common with secondary amenorrhea or hypomenorrhea.
Causes:
In patients with risk factors for endometrial or cervical scarring
(history of uterine or cervical surgery), infections related to IUD use
& severe pelvic inflammatory disease.
Found in 39% patients undergoing hysterosalpingography with
previous postpartum curettage.
Rare cause: Infections (tuberculosis & schistosomiasis).
120. What is the most likely cause for beaded
appearance of fallopian tubes with
clubbed ends of fimbriae on HSG?
a) Genital tuberculosis

b) Chlamydia

c) Neisseria gonorrhoea

d) Endometriosis

Correct Answer - A
Ans: A. Genital tuberculosis
(Ref. Shaw's 16/e p111)
Genital tuberculosis - Most likely cause for beaded appearance of
fallopian tubes with clubbed ends of fimbriae on HSG.
HSG (Hysterosalpingographic) findings:
Tubal occlusion in tuberculosis - Most common sign on HSG.
Occurs most commonly in isthmus & ampulla.
Shows as non-specific finding “Hydrosalpinx”.
Specific pattern like "beaded tube", "golf club tube, "pipestem tube",
"cobble stone tube" & "leopard skin tube"."
Multiple constrictions along fallopian tube courses - Due to scarring.
Hence, 'beaded' appearance to tubes.
Scarring lead to a 'rigid pipe stem' appearance of tubes.
121. Which of these is diagnostic of
menopause?
a) Serum FSH > 40

b) Serum LH > 20

c) Serum FSH < 40

d) Serum estradiol < 30

Correct Answer - A
Ans: A. Serum FSH > 40
(Ref. Shaw’s. 16/e p66. 15/e p62).
Serum FSH > 40 1U/L is diagnostic of menopause.
Criteria for Menopause
Estrogen (E2): 10-20 pg/ml° E2/E1 < 1°
Estrone (El): 30-70 pg/ml° Urine FSH > 40 IU/L°
Laboratory diagnosis:
FSH & estrogen level - Assess ovarian failure.
Especially in premature ovarian failure case or women seeking
treatment for infertility.
FSH levels:
Greater FSH level (>40 m IU/ml).
Documents ovarian failure associated with menopause.
Estrogen level:
Normal/elevated - Depending on stage of menopausal transition.
After menopause extremely low estrogen level.
Evaluated to assess women's response to hormone replacement
therapy.
122. A 23-year-old lady taking antiepileptics
for a seizure disorder gets married.
When should folic acid supplementation
advised to the patient?
a) Any time as soon as she presents to the clinic irrespective of
pregnancy

b) Three months before becoming pregnant

c) 1st trimester

d) As soon as pregnancy is confirmed

Correct Answer - A
Ans: A. Any time as soon as she presents to the clinic irrespective of
pregnancy
If a pregnancy is planned in high-risk women (previously affected
child with neural tube defects), supplementation should be started
with 4 mg (= 4000 microgram) of folic acid daily, beginning 1 month
before the time of the planned conception.
Recommendations:
By U.S. Public Health Service.
Folic acid 0.4 mg daily - For all women of childbearing age & ones
capable of becoming pregnant.
Folic acid 4 mg (= 4000 microgram) daily - For planned pregnancy in
high-risk women (previously affected child) - Beginning 1 month
before time of planned conception.
123. In the pelvic inlet, which is the shortest
anteroposterior diameter?
a) True conjugate

b) Obstetric conjugate

c) Anatomical conjugate

d) Bispinous diameter

Correct Answer - B
Ans: B. Obstetric conjugate
In pelvic inlet shortest anteroposterior diameter - Ostetric conjugate
(10 cm).
Pelvic inlet:
Four diameters:
Anteroposterior, transverse & two oblique diameters.
Anteroposterior diameter:
Distinct with specific landmarks.
Most cephalad - Hence, true conjugate.
Extends from upper- most margin ofthe symphysis pubis to sacral
promontory.
Clinically important obstetrical conjugate is shortest distance
between the sacral promontory and the symphysis pubis. Normally
10 cm or more.
124. Estrogen and progesterone in the first 2
months pregnancy are produced by:
a) Fetal ovaries

b) Fetal adrenal

c) Placenta

d) Corpus luteum

Correct Answer - D
Ans: D. Corpus luteum
(Ref Williams 24/e p169; Ganong 25/e p412, 24/e p414).
Estrogen and progesterone in first 2 months of pregnancy -
Produced by Corpus luteum.
Functions of corpus luteum:
Enlarged corpus luteum of pregnancy secretes estrogens,
progesterone & relaxin.
Progesterone & relaxin -
Helps maintain pregnancy.
By inhibiting myometrial contractions.
Progesterone prevents prostaglandin production by uterus - Stops
contractions.
Corpus luteum function begins to decline after 8 weeks of
pregnancy.
Yet persists throughout pregnancy.
In humans placenta produces sufficient estrogen & progesterone
from maternal & fetal precursor taking over corpus luteum function
after 6th week of pregnancy.
Ovariectomy before 6th week à Hence cause abortion.
Ovariectomy after 6th week doesn’t affect pregnancy.
Note:
hCG secretion decreases after initial marked rise.
Estrogen & progesterone secretion increase until just before
parturition.
125. What is the first sign of puberty in a girl?
a) Thelarche

b) Menarche

c) Adrenarche

d) Pubarche

Correct Answer - A
Ans: A. Thelarche
The first physical sign of puberty in girls is usually a firm, tender
lump under the center of the areola of one or both breasts; occurring
on average at about 10.5 years of age. This is referred to as
thelarche.
Order of Signs of Puberty
Males (TPAM) Females (TPM)
Testicular enlargement (First sign)°
Thelarche (First sign)°
Pubarche°
Pubarche°
Adrenarche°
Menarche°
Moustache & Beard°

Onset of Puberty
Males Females
Growth of testes ( 24 mL in
volume or 2.5 cm in
longest diameter) & thinning of Breast development (thelarche) is
scrotum are first signs of puberty usually first sign of puberty (10-11
(11- 12 year)°. years of age)
These are followed by Followed by the appearance
pigmentation of scrotum & of pubic hair (pubarche) 6-12
months later°.
growth of penis & by pubarche°. Interval to the onset of menstrual
Appearance of axillary hair activity (menarche) is usually 2-2.5
usually occurs in mid-puberty°. years, but may be as long as 6
In males, unlike in females, years°.
acceleration of growth is Peak height velocity occurs early
maximal at genital stages IV-V (at breast stages 11-81 typically
(typically between 13 & 14 years between 11-12 years of age) in
of age)°. girls and always precedes
In males, growth spurt occurs menarche°.
approximately 2 year later than Mean age of menarche is
in females & growth may approximately 12.75 years°.
continue beyond 18 years
of age°.
126. Drug of choice in chronically
hypertensive pregnant women requiring
long term antihypertensive
therapy:
September 2007
a) Nifedipine

b) Metaprolol

c) Methyldopa

d) Hydralazine

Correct Answer - C
Ans. C: Methyldopa
Methyldopa is a drug of first choice for control of mild to moderate
hypertension in pregnancy and is the most widely prescribed
antihypertensive for this indication.
During long term use in pregnancy, methyldopa does not alter
maternal cardiac output or blood flow to the uterus or kidneys, and
for all these reasons it is generally considered the agent of choice
for chronic blood pressure control in pregnancy.
Because the safety record of labetalol in pregnancy is not as well
established as that of methyldopa, labetalol should probably be
considered a second-line agent for pregnant women with chronic
hypertension requiring long term drug therapy
127. A G3P2, pregnant comes to your clinic at
18 weeks of gestation for genetic
counselling. She has a history of two
kids born with thalassemia major. Which
test would you recommend now?
a) Amniocentesis

b) Chorionic villus sampling

c) Cordocentesis

d) Non-invasive prenatal testing

Correct Answer - C
Ans: C. Cordocentesis
(Ref Williams 24/e p300; (Thai 8/e p341-344)
Recommended test – Cordocentesis.
As patient is presenting at 18 weeks - Quick method diagnosing
thalassemia antenatally needed.
Note:
Legal age of abortion is only till 20 weeks.
Fetal blood karyotyping accomplished within 24 to 48 hours.
Significantly quicker than (7- to 10-day turnaround time with
amniocentesis or CVS).
128. Modified BPP consists of:
a) NST with AFL

b) NST with fetal breathing

c) NST with fetal movement

d) NST with fetal tone

Correct Answer - A
Ans: A. NST with AFL
(Ref Williams 24/e p342, 343).
Modified Biophysical profile consists of NST with amniotic fluid
index.
Modified Biophysical Profile:
An abbreviated biophysical profile as first-line screening test by
Clark and coworkers (1989).
A vibroacoustic nonstress test performed twice weekly.
Combined with amnionic fluid index determination.
Amnionic fluid index < 5 cm considered abnormal.
Performed in 10 minutes – Excellent antepartum surveillance
method.
No unexpected fetal deaths.
Modified biophysical profile test:
Predictive of fetal well-being complimentary to biophysical fetal
surveillance – By American College of Obstetricians and
Gynecologists and the American Academy of Pediatrics (2012).
129. Which of these is not a non-
contraceptive use of levonorgestrel?
a) Endometriosis

b) Premenstrual tension

c) Complex endometrial hJperplasia

d) Emergencycontraception

Correct Answer - B
Ans: B. Premenstrual tension
(Ref Williams 24/e p701: Goodman Gilman 12/e p1184, 1190)
Pre-menstrual tension is not a non-contraceptive use of
levonorgestrel.
Therapeutic Uses – Levonorgestrel:
Emergency contraception:
Useful within 120 hours as emergency birth control.
Idiopathic menorrhagia:
Excessively heavy, regular menses in the absence of intracavitary
pathology or coagulopathy.
Menometrorrhagia:
Excessive bleeding in amount prolonged in duration (regular or
irregular intervals).
An alternative delivery system - Protect against endometrial
hyperplasia in women taking SERM.
Management of recurrent pelvic pain secondary to multi- treated
endometriosis.
LNG-IUS is licensed for use in menorrhagia & to provide endometrial
protection to perimenopausal & postmenopausal women on
estrogen replacement therapy.
LNG-IUS beneficial in endometriosis, adenomyosis, fibroids,
endometrial hyperplasia & early stage endometrial cancere (where
the patient is deemed unfit for primary surgical therapy).
130. All of these can be used for post-coital
contraception except:
a) Desogestrel

b) Copper-T

c) Levonorgestrel

d) OCP

Correct Answer - A
Ans: A. Desogestrel
(Ref Williams 24/e p714).
Desogestrel is not used as post-coital contraceptive.
Drugs used for Emergency Contraception
Drug Dose
Levonorgestrel 0.75 mg stat and after 12 hours°
Ethinyl estradiol 50
pg + Norgestrel 0.25 2 tab stat and 2 after 12 hours°
mg
Conjugated
15 mg BD x 5 days
estrogen
Ethinyl estradiol 2.5 mg BD x 5 days
Insertion of an IUCD within maximum period of
5-7 days after accidental unprotected
exposure.
Mifepristone
It prevents implantation but is not suitable for
women with multiple sex partners and for rape
victims
2 tablets (60 mg) to be taken twice at an in
Centchroman interval of 12 hours within 24 hours of
intercourse.
intercourse.
Insertion of an IUCD within maximum period of
5-7 days after accidental unprotected
exposure.
Ulipristal
It prevents implantation but is not suitable for
women with multiple sex partners and for rape
victims.
131. Ulipristal acetate is a/an:
a) GnRH agonist

b) Androgen antagonist

c) Selective estrogen receptor modulator

d) Selective progesterone receptor modulator

Correct Answer - D
Ans: D. Selective progesterone receptor modulator
(Ref Harrison 's 19/e p2391,- Goodman Gilman •12/e p1185)
Ulipristal:
Derivative of 19-norprogesterone.
A selective progesterone receptor modulator (SPRM).
Struturally similar to mifepristone.
Unlike mifepristone, ulipristal is relatively weak glucocorticoid
antagonist.
Moa:
Acts as a partial agonist at progesterone receptors.
132. What is the approved dose of
misoprostol in emergent management of
postpartum hemorrhage?
a) 200 mcg

b) 400 mcg

c) 600 mcg

d) 1000 mcg

Correct Answer - C
Ans: C. 600 mcg
(Ref Williams 24/e p785)
Approved dose of misoprostol in emergent management of
postpartum hemorrhage = 600 pg.
Misoprostol:
Derman (2006) compared a 600 pg oral dose given at delivery
against placebo.
Drug decreased hemorrhage incidence from 12 to 6 percent &
severe hemorrhage from 1.2 to 0.2 percent.
133. What is the best time to give anti-D to a
pregnant patient?
a) 12 weeks

b) 28 weeks

c) 36 weeks

d) After delivery

Correct Answer - B
Ans: B. 28 weeks
(Ref Williams 24/e p312: COGT 11/e p353; FERNANDO ARIAS 4/e
p374).
Best time to give anti-D to a pregnant patient is 28 weeks.
According to American College of Obstetricians and
Gynecologists, 2010:
Anti-D immune globulin is given prophylactically to all Rh D-
negative, unsensitized women at approximately 28 weeks.
Second dose is given after delivery if the infant is Rh D-positive .
134. A 28-year-old patient of
neurocysticercosis develops generalized
peeling of skin all over except palms and
soles starting one month after taking
anti-epileptics. What is the most
probable diagnosis?
a) Fixed drug eruption

b) Pemphigus

c) Steven Johnson syndrome

d) TEN

Correct Answer - D
Ans: D. TEN
(Ref Neena Khanna 4/e p83, 174)
Probable diagnosis is toxic epidermal necrolysis.
Toxic epidermal necrolysis (TEN/ Lyell's syndrome:
Rare, life-threatening skin condition that is usually caused by a
reaction to drugs.
Etiology of Epidermal Necrolysis
Drugs Miscellaneous Idiopathic
Anticonvulsants:
Carbamazepine, phenytoin SSE, GVHD,
barbiturates, lamotrigine Lymphoreticular
Chemotherapeutic agents: malignancies 5% of
sulfonamides, penicillin Infections (Mycoplasma patients
NSAIDs: Butazones, pneumoniae, herpes virus
oxicams Others: Allopurinol, infection)
nevirapine
135. Which type of oral candidiasis does not
presents with white patch?
a) Chronic atrophic candidiasis

b) Chronic hyperplastic candidiasis

c) Chronic mucocutaneous candidiasis

d) Pseudomembranous candidiasis

Correct Answer - A
Ans: A. Chronic atrophic candidiasis
Chronic erythematous (atrophic) candidiasis appears as a red, raw-
looking lesion instead of a white patch seen in all other types.
Erythematous (atrophic) candidiasis:
Appears as a red, raw-looking lesion.
Subtypes of erythematous candidiasis:
Denture-related stomatitis, angular stomatitis, median rhomboid
glossitis & antibiotic-induced stomatitis.
Since they are commonly erythematous/atrophic.
Precede pseudomembrane formation,.
Left when membrane is removed, or arise de novo.
Tongue:
Loss of lingual papillae, leaving a smooth area on tongue.
Occurs on dorsum of tongue in long-term corticosteroids or
antibiotic patient.
But occasionally it can occur after only a few days of using a topical
antibiotic.
This is usually termed 'antibiotic sore mouth/stomatitis' because it is
commonly painful as well as red.
136. What is color of medical oxygen
cylinder?
a) Black body with grey shoulder

b) Grey body with black and white shoulder

c) Black body with white shoulder

d) Grey body with black shoulder

Correct Answer - C
Ans: C. Black body with white shoulder
Ref. Anaesthetic Equipments & Procedures, Practical Approach/p37,
38)
The color of a medical oxygen cylinder is a black body with a white
shoulder.
Color Identification of Medical Gas Cylinders
Pin
Name of gas Body Shoulder
index
White & black
Air Grey° 1, 5°
quarter°
Carbon dioxide (Conc. >7.5%) Grey° Grey° 1, 6°
Oxygen Black White° 2, 5F
Carbon dioxide (Conc. <7.5%) Grey° Grey° 2, 6°
Nitrous oxide Blue° Blue° 3, 5°
Cyclopropane Orange°Orange° 3, 6°
Entonox (50% 02 + N20) Blue & white
Blue° 70
quarter°
Helium Brown° Brown° ?
Mix of oxygen & helium (He
Brown° White° 2, 4°
<80.5%)
Mix of oxygen & helium (He
Mix of oxygen & helium (He
Brown° White° 4, 6°
>80.5%)
Nitrogen Black° Black° 1, 4°

137. What is the most reliable site to measure
core temperature during general
anesthesia?
a) Pulmonary artery

b) Distal esophagus

c) Rectum

d) Tympanic membrane

Correct Answer - D
Ans: D. Tympanic membrane
(Ref Miller's 7/e p1550)
Though pulmonary artery is the gold standard site for core
temperature measurement, esophagus has similar reliability and is
the most commonly used site in the anesthetic practice fior
temperature monitoring.
Core-Temperature Monitoring:
Sites for Core Temperature Measurement
Gold standard site for core temperature
Pulmonary artery
measurement
Tympanic
Most accurate for brain temperature
membrane
Best for brain temperature Nasopharynx
Lower end of
Best site & most commonly used for core body
esophagus
temperature
138. While performing a lumbar puncture, a
snap is felt just before entering into the
epidural space. This is due to piercing of
which structure?
a) Ligamentum flavum

b) Supraspinous ligament

c) Duramater

d) Posterior longitudinal ligament

Correct Answer - A
Ans: A. Ligamentum flavum
(Ref Grays 40/e p729, Snell's 9/e p705)
While performing a lumbar puncture, a snap is felt just before
entering into the epidural space.
This is due to piercing of ligamentum flava.
139.
A pregnant woman with placenta previa
started to bleed as she went into labor. Her
blood pressure was 80/50 mm Hg. A lower
segment caesarean section was planned in
view of acute shock. What type of anesthesia
will you plan for this patient?
a) General anesthesia with IV induction by ketamine

b) Spinal anesthesia up to L4 level

c) General anesthesia with IV induction by propofol followed by


maintenance with fluranes

d) Sedation and epidural analgesia

Correct Answer - A
Ans: A. General anesthesia with IV induction by ketamine
(Ref Williams 24/e p516; Morgan 4/e p197-199)
This patient of placenta previa is in labor and has bled into shock.
She should be delivered by cesarean section under general
anesthesia.
General anesthesia is preferred as it is a more controllable modality
and there is a significant risk of hypotension associated with spinal
anesthesia.
Ketamine is the preferred agent in cases of acute shock.
Stimulates sympathetic system causing tachycardia and
hypertension, so it is intravenous anaesthetic of choice for shock.
140. Which of these is most commonly used
as pre-anesthetic medication?
a) Atropine

b) Promethazine

c) Scopolamine

d) Glycopyrrolate

Correct Answer - D
Ans: D. Glycopyrrolate
(Ref: Miller 7/e p293; KDT 366, 117)
Glycopyrrolate is most commonly used as pre-anesthetic
medication.
An anlicholinergic drug used for reducing secretions in the mouth,
throat, airway, and stomach before surgery.
Used before and during surgery to block certain reflexes and to
protect against certain side effects of some medicines.
141. A 30-year old pregnant woman comes to
your clinic with decreased sleep,
increased appetite and hyperactivity for
2 weeks. A diagnosis of mania is made.
Further probing reveals four episodes of
major depression in the past two years.
What drug will you prescribe to this
patient?
a) Haloperidol

b) Lithium

c) Promethazine

d) Clonazepam

Correct Answer - A
Ans: A. Haloperidol
Haloperidol - Only antipsychotic mentioned in the options which can
be used in pregnant patients suffering from bipolar disorders.
142. Which of the following is not a
component of cognitive triad of Beck?
a) Hopelessness

b) Worthlessness

c) Helplessness

d) Guilt

Correct Answer - D
Ans: D. Guilt
Beck's cognitive triad:
Involves negative thoughts about the self (the self is worthless) , the
world/environment (i.e. the world is unfair, helpless), and the future
(the future is hopeless).
Represents three types of negative thoughts present in depression,
as proposed by Aaron Beck in 1976.
The triad forms part of his cognitive theory of depression.
The triad involves negative thoughts about:
The self (i.e. the self is worthless) .
The world/environment (i.e. the world is unfair, helpless).
The future (i.e. the future is hopeless).
143. A young female on antidepressants
presents to the emergency with altered
sensorium and hypotension. ECG
reveals wide QRS complexes and right
axis deviation. Next best step for the
management of this patient:
a) Sodium bicarbonate

b) Hemodialysis

c) Fomepizole

d) Flumazenil

Correct Answer - A
Ans: A. Sodium bicarbonate
(Ref Harrison 19/e p172)
Typically suggest a diagnosis of tricyclic antidepressant poisoning.
Antidote for TCA poisoning - Sodium bicarbonate.
Administered IV 100 mEq (1-2 mEq/kg), and repeated every few
minutes until BP improves and QRS complexes begin to narrow.
Hemodialysis should not be used since TCAs are highly protein-
bound with large volume of distribution.
144. A 23-year old young boy with
schizophrenia is well maintained on
risperidone for the last 2 months. He has
no family history of the disease. For how
long medication should be continued in
this patient?
a) 5 years

b) 6 months

c) 2 years

d) 12 months

Correct Answer - C
Ans: C. 2 years
(Ref: Niraj Ahtda 7/e p67)
Multiple randomized trials have found that maintenance of
antipsychotic medication reduces the risk of relapse over a period of
up to two years.
145. A child with pervasive developmental
disorder will have all of the following
except:
a) Stereotype behaviour

b) Reduced social interaction

c) Poor language skills

d) Impaired cognition

Correct Answer - D
Ans: D. Impaired cognition
(Ref. Kaplan 106-'107-7107' Viral Ahuja 71e p 16346 )
According to DSM-V, intellectual disability should he ruled out prior
to a diagnosis of pervasive developmental defects.
The diagnostic category pervasive developmental disorders (PDD),
as opposed to specific developmental disorders (SDD) is
characterized by delays in the development of multiple basic
functions including socialization and communication.
146. A male patient of bipolar disorder with
history of 5 episodes of mania and 1
episode of depression in last 8 years,
under control by mood stabilizer, and
manic symptoms appear as he tapered
down the drugs. Which of the following
intervention should be carried out to
improve drug compliance?
a) Psychoeducation

b) CBT

c) Supportive psychotherapy

d) Insight-oriented psychotherapy

Correct Answer - A
Ans: A. Psychoeducation
(Ref: Abuja 7/e p67-68)
Patient symptoms of bipolar disorder are well controlled by
medication – Implies pharmacotherapy is efficient and added
psychotherapy is necessary only to keep the patient compliant to the
medication.
Hence psychoeducation of the patient about adherence to strict
treatment is most crucial psychotherapy modality in preventing
relapse.
147. Identify the nerve in the given diagram
whose palsy causes lateral gaze palsy.

a) A

b) B

c) C

d) D

Correct Answer - B
Ans: B. B
(Ref Gray's 41e p437, 664, 670; 40/e p430; Snells 9/e p544)
Lateral gaze palsy is due to paralysis of lateral rectus, which is
supplied by the abducens nerve.
Structures Marked in the Diagram (cavernous sinus):
A = Oculomotor nerve
B = Abducens nerve
C = Ophthalmic nerve
D = Maxillary nerve
Structures in lateral wall of sinus from above downwards:
Oculomotor (III) nerve.
Trochlear nerve (IV)
Ophthalmic (V1) nerve
Maxillary (V2) nerve
Trigeminal ganglion
Structures passing through centre of sinus (content):
Abducent (V1th) nerve.
Internal carotid artery.
Tributaries of cavernous sinus
From the orbit From the brain From the meninges
Superior ophthalmic
vein° Sphenoparietal
Superficial middle
A branch of sinus°
cerebral vein°
the inferior Frontal trunk of the
Inferior cerebral
ophthalmic middle menin?
veins°
veinQ geal vein
Central vein of retina°

Draining channels of cavernous sinus
Into the transverse
sinus through petrosal sinus° Into
Into the pterygoid plexus through
the internaljugularvein through inferior veins°
Into the facial vein through ophthalmic
petrosal sinus° vein°


148. The nerve supplying the superior oblique
muscle exits the brainstem at which of
the following sites:

a) A

b) B

c) C

d) D

Correct Answer - B
Ans: B. B
(Ref: Gray’s 41e 1)664, 670, 40/e. p276)
Superior oblique muscle is supplied by the fourth cranial nerve
(Trochlear nerve).
Only nerve that exits the brainstem from its posterior aspect.
Structures Marked in the
A Oculomotor nerve
B Trochlear nerve
C Abducens nerve
D Facial nerve

Cranial Nerves Location
Cribriform
Olfactory nerve
plate (CN I)
Optic canal (optic nerve, ophthalmic artery, central
retinal vein)
Middle cranial Superior orbital fissure [oculomotor, trochlear,
fossa (CN II- Ophthalmic (V1) nerve abducens, ophthalmic vein,
VI) sympathetic fibers]
sphenoid bone Foramen rotundum [Maxillary (V2) nerve
Foramen rotundum [Maxillary (V2) nerve
Foramen spinosum (middle meningeal artery)
Internal auditory meatus (Facial & Vestibulocochlear
Posterior
nerve)v
cranial fossa
Jugular foramen (Glossopharyngeal, vagus, spinal
(CN VII-XII)
accessory nerve, internal jugular vein)`
through
Hypoglossal canal (Hypoglossal nerve)Q
temporal or
Foramen magnum (Spinal roots of spinal accessory
occipital bone
nerve, brainstem & vertebral arteries.

149. The nerve supplying the superior oblique
muscle exits the brainstem at which of
the following sites:

a) A

b) B

c) C

d) D

Correct Answer - B
Ans: B
Superior oblique muscle is supplied by CN IV i.e. Trochlear nerve
and it is the only CN that exits from dorsal aspect of brain stem
150. Identify the vagus nerve in this given
cross-sectional diagram of the neck:

a) A

b) B

c) C

d) D

Correct Answer - C
Ans: C. C
(Ref Gray's 41/e p982, 40/e p438)
In given diagram:
Vagus descends vertically in neck in carotid sheath.
Between internal jugular vein & internal carotid artery to upper
border of thyroid cartilage.
Continues passing between internal jugular vein & internal carotid
artery to neck roo.
Structures Marked in the Diagram
A Carotid artery
B Internal jugular vein
C Vagus nerve
C Vagus nerve
D Sympathetic trunk
151. All of the following are true about basal
electrical rhythm of intestines except:

a) Tone of contraction is related to amplitude of the stimulus

b) Tone of contraction is related to frequency of stimulation

c) Frequency of contraction is 6 per minute

d) Threshold of contraction is —50 mV

Correct Answer - A
Ans: A. Tone of contraction is related to amplitude of the
stimulus
(Ref Ganong 25/e p496-497, 24/e p498-499)
Tone of contraction depends on stimulus frequency rather than
amplitude.
Threshold for action potential:
Upto 50 mV (I.e. potential required to trigger an action potential &
contraction).
One contraction in 10 seconds implies frequency is 6
contractions/minute.
152. The given graph likely depicts which of
the following disease?

a) Bronchial asthma

b) Emphysema

c) Interstitial lung disease

d) Normal study

Correct Answer - C
Ans: C. Interstitial lung disease
* Pressure-volume loop - Compliance curve.
* Shows decreased slope - Implying decreased compliance (AV/AP).
* Typically seen in restrictive lung diseases & interstitial lung
disease.
* Emphysema - Obstructive lung conditions & increases compliance.
Static Expiratory Pressure-Volume Curves of Lungs
Shift Shift Compliance
Pulmonary edema
Downward & to Compliance
Interstitial pulmonary
right is decreased
fibrosis
Compliance is
Upward & to left Emphysema
increased

153. The following microscopic appearance is
that of schwannoma—a nerve sheath
tumor most commonly involving the
cerebellopontine angle. What does the
area marked with arrow represent?

a) Myxoid tissue

b) Antony A pattern

c) Schwannoma Antoni type B

d) Antony C pattern Verocay body

Correct Answer - C
Ans: C. schwannoma Antoni type B
(Ref Robbins 9/e p257. 854, 8/e p1340).
Area marked with arrow represents growth pattern.
Here tumor is less densely cellular & consists of loose meshwork of
cells, microcysts and myxoid stroma, Antoni B pattern of growth.
Schwannoma - Histopathology:
Verocay bodies- “Nuclear-free zones" of processes lie between
regions of nuclear palisading.
In Antoni Agrowth pattern:
Elongated cells with cytoplasmic processes are arranged in fascicles
in areas of moderate to high cellularity and scant stromal matrix.
In Antoni B growth pattern:
Tumor less densely cellular & consists of a loose meshwork of cells,
microcysts and myxoid stroma.
154. A 12-year-old boy presents with
enlargement of bilateral cervical lymph
nodes for 2 years. A biopsy from the
lymph node reveals the following picture.
Which of the following is the correct set
of diagnosis, etiology and the cell
described?

a) Hodgkin's lymphoma; Epstein Barr Virus and Reed Sternberg


cells

b) Hodgkin's lymphoma; Epstein Barr Virus and Embryo cell

c) Non-Hodgkin's lymphoma; HIV and Giant B cell

d) Tuberculosis, Mycobacterium and Tiny granuloma

Correct Answer - A
Ans: A. Hodgkin's lymphoma; Epstein Barr Virus and Reed
Sternberg cells
(Ref: Robbins 9/e p608, 8/e p617)
(Ref: Robbins 9/e p608, 8/e p617)
Given slide demonstrates Reed-Sternberg cells typical of Hodgkin's
Lymphoma.
EBV infection - Risk factor for Hodgkin's Lymphoma.
Reed-Sternberg cells & variants - Identification:
Diagnostic Reed-Sternberg cells:
Large cells (45 pm in diameter) with multiple nuclei or a single
nucleus with multiple nuclear lobes, each with a large inclusion-like
nucleolus about the size of a small lymphocyte (5 to 7 pm in
diameter).
155. A liver biopsy reveals following findings.
What is true about this condition?

a) Nutmeg liver with dark areas of perivenular dead hepatocytes


and gray areas of periportal viable hepatocytes.

b) Nodular regenerative hyperplasia of liver induced due to OC Ps.

c) Nutmeg liver with pale areas of necrosis and dark congested


areas of perivenular viable hepatocytes.

d) Cirrhotic liver with fibrotic nodules.

Correct Answer - A
Ans: A. Nutmeg liver with dark areas of perivenular dead
hepatocytes and gray areas of periportal viable hepatocytes
Ref Robbins 9,/e p864, 8/e p872)
Nutmeg liver - Classical picture:
Seen in chronic passive venous congestion on liver.
Dark congested areas - Necrotic hepatocytes.
Surrounding paler & brownish appearing regions - Viable
hepatocytes.
Centrilobular hemorrhagic necrosis - Due to combined
hypoperfusion & retrograde congestion acting synergistically.
Variegated mottled appearance of liver - Reflecting hemorrhage
& necrosed centrilobular regions.
Referred as “nutmeg liver” - Due to its resemblance to cut
surface of nutmeg.
156. A 30-years-old male patient presents
with a history of heartburn. An upper GI
endoscopy was done and a biopsy from
a suspicious lesion showed the following
picture. What does this show? Which
special stains will you do for
confirmation and what will you look for?

a) H. pylori infection, Silver stain, Gram-negative coccobaci Ili

b) Adenocarcinoma esophagus, mucin stain. mitotic figures

c) Squamous cell carcinoma, keratin stain

d) Barrett's esophagus, mucin stain, dysplasia


Correct Answer - D
Ans: D. Barrett's esophagus, mucin stain, dysplasia
Ref Robbins 9/e p757, 758, 8/e p771
Image suggestive of Barrett's esophagus.
Extent of dysplasia is better delineated with mucin stain to rule out
malignant trait.
Goblet cells:
Metaplastic specialized (intestinalized columnar) epithelium in place
of distal squamous mucosa.
Also called “columnar lined esophagus”.
Occur as response to chronic injury.
Regresss after treatment.
157. Cell growth can be stopped at some
specific checkpoints in the cell cycle as
depicted. Which stage of cell cycle
demonstrates the primary point in
regulation of cell growth?

a) End of M

b) End of G1

c) End of S

d) End of G2

Correct Answer - B
Ans: B. End of G1
(Ref Robbins 9/e p289, 8/e p286)
G1-S:
End of G1 phase - Primary & most important cell cycle
checkpoint.
Both cell cycle regulators p53 & RB gene act here.
158. For what procedure is the instrument
depicted in the diagram used?

a) Bone marrow biopsy.

b) Pleural biopsy.

c) Kidney biopsy.

d) Liver biopsy.

Correct Answer - A
Ans: A. Bone marrow biopsy.
This is Argon bone marrow biopsy needle
Various types of bone marrow biopsy needles presented here.
159. III produce an enzyme that blocks the
action of beta lactam antibiotics in
periplasmic space. Which arrow in the
structural diagram of Penicillin G
denotes the site of action of this
enzyme?

a) A

b) B

c) C

d) D

Correct Answer - B
Ans: B. B
(Ref: Goodman p1478; Katzung 13/e p770, KDT p /6, 6/c p694)
Beta-lactam:
An amide bond.
Found in Penicillin G structure is represented by 'B’.
Beta lactam ring:
Fused to five-membered thiazolidine ring.
Fusion causes β-lactam ring to be more reactive (than monocyclic β-
lactams).
Two fused rings distort β -lactam amide bond à Removes resonance
stabilization normally found in these chemical bonds.
160. A guinea pig is dissected and a portion
of its intestine is fixed in the modified
Dales chamber to study the effects of
some drugs on intestinal contractility. A
substance X was infused in the broth
following which the graph comes out as
depicted. Substance X most closely
resembles which of these substances?

a) Epinephrine

b) Acetylcholine

c) KC1

d) BaCl2

Correct Answer - A
Ans: A. Epinephrine
(Ref: Goodman Gilman 12/e p188, 285: Katzung 13/e p89, 12/e p90;
KDT 7/e p132, 6/e p120)
Graph showng decreased contractility after infusion of substance X.
Intestinal muscles show increase in contractility on action of
acetylcholine on M3 receptors whil
epinephrine inhibits contraction.
Hence, the substance X is likely to be a sympathomimetic drug, i.e.
Epinephrine.
161. Stool examination in a patient reveals the
following finding. What is the likely route
of infection of this parasite?

a) Ingestion of food contaminated with the egg of larva

b) Insect bite

c) Improperly cooked beef

d) Swimming in dirty water pool

Correct Answer - A
Ans: A. Ingestion of food contaminated with the egg of larva
Elongated, barrel-shaped eggs with a polar 'plug' at each end shown
here are characteristic of Trichuris egg, which causes trichuriasis via
frco-oral transmission.
Nematodes Fecal examination
Trichinella spiralis Adult worm°
Trichuris trichiura Eggs (barrel shaped)°
Strongyloides Rhabditiform larvae°
Ancylostoma duodenale & Egg which may be hatched, so
Necator americanus rhabditiform larvae can be seen
Enterobius vermicularis Usually not useful
Ascaris lumbricoides Eggs & adult worm°
Ascaris lumbricoides Eggs & adult worm°
Filariasis No role
Dracunculus medinensis No role
162. The following picture was seen in nasal
biopsy from a patient with brain abscess.
Identify the organism seen and the stain
used?

a) Staphylococcus, Gram-stain

b) Streptococcus, Gram-stain

c) Cryptococcus, India ink

d) Nocardia, Gram-stain

Correct Answer - D
Ans: D. Nocardia, Gram-stain
(Ref Ananthanarayan 10/e p400, 8/e p393; Harrison 19/e p1085,
1086)
Nocardia species - Shows branching pleomorphic rods.
Acid fast.
Can be stained by Modified Ziehl-Nieelsen technique.
163. A smear was prepared from the genital
ulcer. Identify the organism responsible:

a) Chlamydia

b) Treponema pallidum

c) Trichomonas vaginalis

d) Neisseria gonorrhea

Correct Answer - C
Ans: C. Trichomonas vaginalis
(Ref Paniker c Parasitology 7/e p34, 6/e p4I; Jawetz 27/e p713).
Given smear shows infestation of Trichomonas vaginalis.
164. A 24-year-old female presented with an
ulcer in the genital area. A Giemsa
stained cervical smear was taken which
showed the following image. Identify the
causative agent:

a) Chlamydia

b) Gardnerella vaginalis

c) Hemophilus ducreyi

d) Calymmatobacterium donovani

Correct Answer - D
Ans: D. Calymmatobacterium donovani
(Ref Ananthanarayan 10/e p404, 8/e p396, Jaunt: ?7/e p761,
Harrison 19fe p198e-1)
Coccobacilli seen in the Giemsa-stained specimen suggest a
diagnosis of granuloma inguinale.
Donovan bodies:
Seen intracellularly in mononuclear cells.
Calymmatobacterium donovani:
Minute, encapsulated, coccobacilli.
Closely related to Klebsiella genus.
Sexually transmitted.
Causes Granuloma inguinale/ donovanosis - chronic inflammatory
disease.
165. A patient presented with headache and
projectile vomiting along with alteration
in sensorium. The following parasite
demonstrated on India ink staining. What
is the likely diagnosis?

a) Coccidioides

b) Histoplasma

c) Blastomyces

d) Cryptococcus

Correct Answer - D
Ans: D. Cryptococcus
Ref: Ananthanarayan p6I6, 8/e p611; Harrison 19/e p1340, Jawetz
27/e p687-688)
The diagram above shows capsule of Cryptococcus stained with
India ink.
Alternate staining methods are nigrosin and mucicarmine
Cryptococcosis diagnosis:
Requires the demonstration of yeast cells in normally sterile tissues.
Visualization of the capsule of fungal cells in cerebrospinal fluid
(CSF) mixed with India ink is a useful rapid diagnostic technique.
Cryptococcal cells in India ink have a distinctive appearance
because their capsules exclude ink particles.
166. This is a schematic diagram depicting
body structure of which of these
helminths?

a) Onchocerca volvulus

b) Brugia malayi

c) Loa loa

d) Wuchereria bancrofti

Correct Answer - B
Ans: B. Brugia malayi
(Ref: Paniker's Parasitology 7/e p214, 6/e p2031)
The microfilariae given in the question has morphology typical of
Brugia malayi.
Brugia malayi:
Head space is twice as long as it is broad, nuclei in body appears
crowded & tail tapers around to two nuclei that appears to be
connected by a fine thread.
167.
A patient presents with progressive
dyspnoea. Chest X-ray revealed a cavitary
lesion in lower lobe of right lung. The
following histopathological appearance was
seen of resection of the cavity. What is the
most likely cause of this disease and the
number of layers that can be seen in the wall
of the parasite?

a) Cysticercosis with 3 layers

b) Strongyloides with 2 layers

c) Echinococcus with 2 layers

d) Paragonimus with 2 layers

Correct Answer - C
Ans: C. Echinococcus with 2 layers
(Ref Panikers Parasitology 7/e p127, 6/e p153; Jawetz 27/e p736;
Sahiston 20/e p1452, 19/e p1447-1449; Blumgart 5/e p1035-1048:
Shackelford 7/e p1459-1462)
The histopathology image given is typical of hydatid cyst, caused by
Echinococcus granulosus.
Cysts of Echinococcus:
Enclosed by inner, nucleated, germinative layer & outer, opaque,
non-nucleated layer.
168. What is the name of the disease
transmitted by this vector?

a) Filaria

b) Leishmaniasis

c) Yellow fever

d) Malaria

Correct Answer - B
Ans: B. Leishmaniasis
The given image is of sand fly (hairy body and erect wing). Sandfly
is the primary vector of leishmaniasis and pappataci fever.
Description of Sandfly
Adults are about 1.5-3.0 mm long and yellowish, with
conspicuous black eyes, hairy
bodies, wings, & legs°.
Adults The oval lanceolate wings are carried erect on the humped
AdultsThe oval lanceolate wings are carried erect on the humped
thorax.
Males possess long prominent genital Terminalia known as
claspers. Females have a pair of anal recti.
Only female sandflies can bite in the dwelling at night.
Bites It takes shelter during the day in holes & crevices in the wall,
in dark room & storeroom, etc.
Life
The average life of a sandfly is about 2 weeks.
Span

169. Identify the disease in which the
following treatment is used?

a) Influenza

b) Kala-Azar

c) Tuberculosis

d) Leprosy

Correct Answer - D
Ans: D. Leprosy
The given image is of the multibacillary adult blister pack, which is
used in multibacillary leprosy.
WHO has designed blister pack medication kits for both
paucibacillary leprosy and multibacillary leprosy.
Each easy to use kit contains medication for 28 days.
Blister pack medication kit for single lesion paucibacillary leprosy
contains the necessary medication for the one-time administration of
the 3 medications.
170. The following life cycle most closely
resembles that of which of the following
viruses?

a) Influenza A

b) Dengue

c) Japanese encephalitis virus

d) Kyasanur forest disease virus

Correct Answer - C
Ans: C. Japanese encephalitis virus
It is clear from the given diagram below that the lifecycle given in the
question is that of the Japanese Encephalitis Virus.
Humans are dead-end and incidental hosts in the lifecycle of these
viruses.
171. The following box plot shows the
distribution of three sets of data around
the mean. What is the correct sequence
of inference from this box plot?

a) 1-Normal distribution, 2-Positive skewed, 3-Negative skewed

b) 1-Normal distribution, 2-Negative skewed, 3-Positive skewed

c) 1-Negative skewed, 2-Positive skewed, 3-Normal distribution

d) I -Positive skewed, 2-Normal distribution, 3-Negative skewed

Correct Answer - B
Ans: B. 1-Normal distribution, 2-Negative skewed, 3-Positive
skewed
(Ref http://wwwphysics.csb,spi.edu/stats/box2.hoirh
Positive or negative skewed data:
Defined by the direction of the tail, i.e. direction of the least
frequency values.
Similarly in this box plot we can see that the data is equally
distributed on either sides of the mean box in Plot (1).
In Plot (2), the median is towards the higher side and most values
are distributed towards the higher side hence it is negatively
skewed.
Hence, vice versa for Plot (3).
172. The following scatter plot of 4 different
samples shows the correlation between
weight and height in the samples. All 4
samples have the same coefficient of
correlation of 0.6 taken together, what
will be the net correlation coefficient?

a) More then 0.6

b) Less then 0.6

c) 0.6

d) Cannot be calculated

Correct Answer - C
Ans: C. 0.6
Coefficient of correlation is given by the slope of the graph, since the
slope of all four populations are same i.e. 0.6, overall correlation
coefficient will remain as 0.6.
173. What is following type of data
description called?

a) Stern and leaf diagram

b) Box whisker plot

c) Forrest plot

d) Funnel plot

Correct Answer - A
Ans: A. Stern and leaf diagram
Stem and Leaf Plot:
Special table where each data value is split into a 'stem' (the first
digit or digits) and a 'leaf' (usually the last digit).
Stem-and-leaf display:
Device for presenting quantitative data in a graphical format, similar
to a histogram, to assist in visualizing the shape of a distribution.
Contains two columns separated by a vertical line.
Left column contains the stems.
Right column contains the leaves.
174. A 65-year-old patient presents to your
clinic with a history of chest pain for last
24 hours associated with sweating and
diaphoresis. The following ECG findings
are seen. BP is 150/90 mm Hg. Which of
the following is not given in management
of this patient?

a) Thrombolysis

b) Aspirin

c) Statin

d) Morphine

Correct Answer - A
Ans: A. Thrombolysis
Symptoms and ECG findings are suggestive of ST-elevation MI.
Since the patient has come to the hospital 24 hours after onset of
symptoms, there will be no benefit from thrombolysis.
Management of Patients with ST-segment Elevation Myocardial
Infarction:
Prehospital, emergency medical service providers should administer
aspirin (162 to 325 mg) to all patients not already taking aspirin,
obtain a 12-lead electrocardiogram (ECG) in patients suspected of
having STEMI, review a reperfusion checklist, and relay this
information to a medical facility.
Patients with STEMI who have cardiogenic shock, those with
contraindications to lytics, and those at high-risk of dying because of
heart failure should be channeled immediately to a facility capable of
cardiac catheterization.
175. A following ECG was recorded from a
patient presenting to the emergency.
What is the likely diagnosis?

a) Sinus arrhythmia

b) Atrial Fibrillation

c) PSVT

d) Heart block

Correct Answer - B
Ans: B. Atrial Fibrillation
The ECG shows absent P waves with baseline variation.
This is seen in atrial fibrillation.
ECG in AF:
Characterized by the lack of organized atrial activity and irregularly
irregular ventricular response.
176. Which of the following drugs is not
indicated in the management of the
condition seen in the following.

a) Metaprolol

b) Adenosine

c) Amiodarone

d) Diltiazem

Correct Answer - B
Ans: B. Adenosine
(Ref Harrison 191E, p1480 184,7)1882)
* The given ECG is showing atrial fibrillation.
* Adenosine is used in the treatment of PS VT, not in atrial fibrillation
- β-blockers (Metaprolol) can be used to reduce the ectopics that
initiate AF and are the usual first line therapy.
- Amiodarone is also effective prevents episodes.
- Blood thinners- Apixaban (Eliquis)
- Aspirin
- Clopidogrel (Plavix)
- Dabigatran (Pradaxa)
- Enoxaparin (Lovenox)
- Heparin
- Rivaroxaban (Xarelto)
- Most common way to treat atrial fibrillation is with drugs that
control your heartbeat-
- Metoprolol (Lopressor, Toprol)
- Propranolol (Inderal, Innopran)
- Timolol (Betimol, Istalol)
- Calcium channel blockers-
- Diltiazem (Cardizem, Dilacor)
- Verapamil
- Potassium channel blockers, which slow the electrical signals that
cause AFib-
- Amiodarone (Cordarone, Nexterone Pacerone),
- Dofetilide (Tikosyn)
177. Identify this crystal found in u urine
analysis:

a) Calcium carbonate stone

b) Ammonium phosphate stone

c) Uric acid

d) Calcium oxalate stone

Correct Answer - D
Ans: D. Calcium oxalate stone
(Ref Smith 17/e p249-254: Campbell 10/e p1296-1302)
In the given image, which shows enveloped or bipyramidal crystals
are seen in calcium oxalate (dihydrate) stones.
178. A 17-year-old patient develops
intussusception for which he was
operated and a segment of intestine
showing multiple polyps was resected.
Microscopy showed the following
pathology. What is the likely diagnosis?

a) Tubulovillous polyps

b) Hamartomatous polyps

c) Inflammatory polyps

d) Adenocarcinoma

Correct Answer - B
Ans: B. Hamartomatous polyps
(Ref Sabiston 20/e p1372, 19/e p1342; Schwartz 10/e p1206 9/e
p1042-104:3; Bailey 27/e p12.50. 26/e p1161-1164)
The given histology shows a large and pedunculated polyp with a
firm lobulated contour.
An arborizing network of connective tissue is seen with well-
developed smooth muscle extending into the polyp and surrounding
normal abundant glands lined by normal intestinal epithelium rich in
goblet cells.
This is typical of Peutz-Jegher's polyps, which are a type of
hamartomatous polyp.
179. A28-year-old female patient presented
with lower abdominal pain along with
dysmenorrhea. The following finding was
seen on laparoscopic examination. What
is the likely diagnosis?

a) Krukenberg tumor

b) Polycystic ovaries

c) Endometriosis

d) Cystadenoma of ovary

Correct Answer - C
Ans: C. Endometriosis
(Ref Shaw's 16/e p413-414, 157e p471,.Novaks 13/e p3662.
Highly suggestive of endometriosis.
Chocolate Cyst:
Caused by endometriosis.
Formed by tiny patch of endometrial tissue.
Mucous membrane building on inner uterine wall layer→ bleeds,
sloughs off, → transplanted & grows → enlarges inside ovaries.
On blood building up → turns brown.
On rupture → material spills over into pelvis & onto surface of
uterus, bladder, bowel & corresponding spaces in between.
180. Following fetal tocographic finding was
seen in a 30-year- old female patient in
labor. What does it suggest?

a) Early cord compression

b) Fetal distress

c) Head compression

d) Fetal anemia

Correct Answer - B
Ans: b.�Fetal distress
(Ref: Williams 24/e p340)
Late decelerations, typical of fetal distress can be seen in the given
fetal tocography.
Cardiotocography (CTG):
Interpretation:
Includes description of uterine activity (contractions), baseline fetal
heart rate, baseline FHR variability, presence of accelerations,
periodic or episodic decelerations
Based on,
3 primary mechanisms by which uterine contractions can cause a
decrease in fetal heart rate are compression of fetal head, umbilical
cord & uterine myometrial vessels.
181. HSG image below shows:

a) Endometrial polyp

b) Genital TB

c) Fibroid uterus

d) Asherman syndrome

Correct Answer - A
Ans: A. Endometrial polyp
(Ref: .Shaw's 16/e p103)
The HSG shows a well-defined filling defect characteristically seen
in endometrial polyps.
Endometrial Polyp:
Diagnosis:
Ultrasound:
Echogenic, smooth, intracavitary masses outlined by the fluid (well-
defined, homogenous, polypoid isoechoic lesion).
Hysterosalpingogram:
Pedunculated or sessile-filling defects within the uterine cavity.
182. Identify the following congenital defect:

a) Spina bifida occulta

b) Dermoid cyst

c) Meningocele

d) Cystic hygroma

Correct Answer - C
Ans: C. Meningocele
(Ref. Ghai 8/e p576)
The given image shows a skin covered, brilliantly transilluminant
defect in the back in a newborn.
This is typical of a meningocele.
Meningocele - Clinical Features:
A fluctuant midline mass that may transilluminate occurs along the
vertebral column, in the lower back.
Most meningoceles are well covered with skin and pose no threat to
the patient.
Anterior meningocele:
rojects into pelvis through a defect in sacrum.
Symptoms of constipation & bladder dysfunction develop.
Due to increasing lesion size.
183. Identify the deformity seen in the given
picture:

a) Hallux valgus

b) Hallux varus

c) Cubitus valgus

d) Rheumatoid nodule

Correct Answer - A
Ans: A. Hallux valgus
(Ref: Apley’s 603, 604)
The given picture shows hallux valgus in which outward or lateral
deviation of great toe occurs.
Hallux valgus:
Commonest of the foot deformities (and probably of all
nzusculoskeletal deformities).
In people never worn shoes the big toe is in line with the first
metatarsal, retaining the slightly fan- shaped appearance of the
forefoot.
In people who habitually wear shoes the hallux assumes a valgus
position.
But only if the angulation is excessive is it referred to as `hallux
valgus'.
184. Which of the following test is being
demonstrated by the given image?

a) Straight leg raising test

b) Thomas test

c) Narath sign

d) Trendelenburg test

Correct Answer - B
Ans: B. Thomas test
(Ref Apley's 9/e p495)
Given image demonstrates Thomas test.
Both hips are flexed simultaneously to their limit, thus completely
obliterating the lumbar lordosis.
Holding the 'sound' hip firmly in position à hence, keeping pelvis still
à Other limb lowered gently à with any flexion deformity the knee will
not rest on couch.
185. A 9-year-old child presents to your clinic
with the following deformity. Which is
the most likely fracture leading to such a
defect?

a) Colle's fracture

b) Lateral epicondyle fracture

c) Medial epicondyle fracture

d) Supracondylar fracture

Correct Answer - D
Ans: D.�Supracondylar fracture
(Ref: Apleys 9/e p371, 760)
The given picture shows cubitus varus or gun-stock deformity.
The most common cause is ma/union of a supracondylar fracture.
Cubitus Varus (`Gun-Stock' Deformity):
The deformity is most obvious when the elbow is extended and the
arms are elevated.
The most common cause is malunion of a supracondylar fracture.
The deformity can be corrected by a wedge osteotomy of the lower
humerus but this is best left until skeletal maturity.
186. A child presented with pain in the
forearm following a trauma. An AP and
lateral X-ray of the forearm reveal the
findings as shown. What is the most
likely diagnosis?

a) Colle's fracture

b) Smith fracture

c) Monteggia fracture

d) Galeazzi's fracture

Correct Answer - D
Ans: D. Galeazzi's fracture
(Ref Apley's 9/e p771)
In the given X-ray, a transverse or short oblique fracture is seen in
the lower third of the radius, with angulation or overlap.
The distal radio-ulnar joint is subluxated or dislocated.
Classically seen in Galeazzi's fracture.
Galeazzi's fracture's X-ray:
A transverse or short oblique fracture is seen in the lower third of the
radius, with angulation or overlap.
The distal radio-ulnar joint is subluxated or dislocated.
187. A lady presents with history burning
sensation on eating spicy food. On
examination bilateral white lacy streaks
are present in buccal mucosa. There is
no history tobacco abuse but
amalgamated third molar is present.
What is the diagnosis?

a) Leukoplakia

b) Lichen planus

c) Candidiasis

d) Aphthous stomatitis

Correct Answer - B
Ans: B. Lichen planus
(Ref Fitzpatrick 7/e p244-255; Rooks 8/e p41.1-41.20; Roxburgh
18/e p154-158)
History of burning sensation on eating spicy food and on
examination bilateral white lag, streaks (Wickham’s striae) in buccal
mucosa and amalgamated third molar is highly suggestive of Lichen
planus.
Lichen planus:
Self-limiting papulosquamous inflammatory disorder of skin of
unknown origin.
Typical lesion:
Pruritic, polygonal, purple, plain topped papule & plaques which
often has whitish lacy pattern on its surface (Wickham's striae).
188. A young male presents with the
following itchy lesion for one month. All
of the following genera can cause this
kind of lesion except:

a) Aspergillus

b) Trichophyton

c) Microsporum

d) Epidermophyton

Correct Answer - A
Ans: A. Aspergillus
(Ref: Neena Khanna 4/e p2R5)
The lesions in the given picture are suggestive of Tinea cruris.
Also known as Jack itch.
Dermatophytosis of the groin region caused by usually Trichophyton
rubrum (not the aspergillus).
Dermatophytes:
Group of fungi (ring worm), that under moist conditions have the
ability to infect & survive only on dead keratin, that is the superficial
topmost layer of skin, hair & nails.
Belongs to 3 genera Microsporum , Epidermophyton, & Trichophyton
189. An 18-year-old female presents to you
with a hypopigmented patch on the
medial aspect foot with feathery margin.
The rest of the physical examination is
normal. Which of the following drug
should not be used in the treatment of
this patient?

a) Topical tacrolimus

b) Clobetasol

c) Isotretinoin

d) PUVA

Correct Answer - C
Ans: C. Isotretinoin
(Ref Fitzpatrick 7/c p624-625, Rooks 8/e p58.39-58.51: Harrison
(Ref Fitzpatrick 7/c p624-625, Rooks 8/e p58.39-58.51: Harrison
19/e p.do -359,18/e p409-412)
In the given picture, patient is having localized vitiligo.
Isotretinoin is used in management of acne and it should always he
avoided in a young female because of teratogenic potential.
Drugs used in treatment of Vitiligo:
Topical clobetasol is a topical steroid
Topical tacrolimus is an immunosuppressant
Topical methoxasalen is a psoralen
190. A young boy with oily skin presents with
acne as shown. What is the appropriate
treatment?

a) Oral isotretinoin

b) Oral steroid

c) Topical retinoic acid

d) Benzoyl peroxide

Correct Answer - A
Ans: A. Oral isotretinoin

In the given picture patient is suffering from acne with pus filled
lesions.
Oral therapy with Isotretinoin is indicated is such a patient.
Isotretenoin:
Used for nodulocystic acne / severe acne.
Side effects:
Rash (MC)
Teratogenic (contraindicated in pregnancy)
Hypertriglyceridemia (on prolonged use).
191. A 28-year-old lady has asymptomatic
dome shaped small lesions on forehead
for last 2 months. She has a 2-year- old
daughter with similar lesions. What is the
causative agent of the lesi -1?

a) Papillomavirus

b) Poxvirus

c) Herpes virus

d) Coxsackie virus

Correct Answer - B
Ans: B. Poxvirus
(Ref Neena Khanna 4/e p274)
The lesions in the given picture on the forehead are pearly white
papules, with a history of transmission to daughter, most likely to be
Molluscum contagiosum, which is caused by the poxvirus.
192. A young male presented with anesthetic
patch on right forearm. A thickened
nerve was palpable on examination. Skin
biopsy shows the following picture. What
is the diagnosis?

a) TT

b) LL

c) Histiocytosis

d) Lymphoma

Correct Answer - A
Ans: A. TT
(Ref Neena Khanna 4/e p266)
The patient has presented with a single anesthetic patch and single
enlarged nerve, and a well-defined granuloma can be seen on the
skin biopsy.
These findings are typical of Tuberculoid leprosy.
193. The given instrument is used for?

a) Pleural biopsy

b) Lumbar puncture

c) Liver biopsy

d) Bone marrow biopsy

Correct Answer - B
Ans: B. Lumbar puncture
The given needle in the question is used for lumbar puncture.
194. Identify the use of the following
instrument:

a) Laryngoscopy

b) Bone marrow aspiration

c) Intraoperative retraction

d) None of thesea Carina

Correct Answer - A
Ans: A. Laryngoscopy
(Ref: Miller 7'e o1587: Lee 13/e p210)
This is an image of a laryngoscope with curved blade. Laryngoscope
is used in direct laryngoscopy.
Laryngoscope Blades:
Commonly used straight blade: Magill Bladee (used in infants).
Commonly used curved blade: Macintosh bladee
195. The diagram of a proseal-type Laryngeal
Mask Airway is provided below. Above
what site is the arrow marked area of the
airway positioned?

a) Carina

b) Upper end of trachea

c) Vocal cords

d) Above esophagus

Correct Answer - D
Ans: D. Above esophagus

The arrow in the given picture shows the internal gastric tube
(drainage tube) kept just above the esophageal opening to aspirate
GI secretions.
LMA Pro-Seal:
Reusable supraglottic airway device that incorporates a gastric
drainage tube placed lateral to the main airway tube and which ends
at the tip of the mask.


The gastric drainage tube is designed to separate the
gastrointestinal and respiratory tracts, allowing regurgitated fluid to
pass up the drain tube and bypass the glottis, thereby protecting the
airway from soiling in the event of passive regurgitation.
196. A patient was in surgery when suddenly
a resident noticed the following plot on
the capnograph monitor. What should be
the immediate next step in manage​ment
of the patient?

a) Increase muscle relaxant

b) Increase the depth of anesthesia

c) Check tubes for any blockage

d) Change soda-lime

Correct Answer - A
Ans: A. Increase muscle relaxant
A cleft in capnograph is due to patients' own respiratory effort
because of inadequate muscle relaxation.
Increasing the dose of muscle relaxant will solve this problem.
197. A 20-year old male patient present with
pain movement. X-ray of knee joint
shows lytic lesion on the upper end of
tibia. What is the likely diagnosis?

a) Simple bone cyst

b) Adamantinoma

c) Multiple myeloma

d) Osteoclastoma

Correct Answer - D
Ans: D. Osteoclastoma
(Ref Apley 9/e p202)
Given history & X-ray with epiphyseal lytic lesion in tibia with a `soap
bubble' appearance is characteristic of osteoclastoma.
The tumour always abuts against the joint margin in osteoclastoma.
X-ray findings of Giant-cell Tumour:
Radiolucent area situated eccentrically at the end of a long bone &
bounded by subchondral bone plate.
The centre sometimes has a soap-bubble appearance due to ridging
of the surrounding bone.
Appearance of a 'cystic' lesion in matue bone, extending right up to
the subchondral plate, is so characteristic that the diagnosis is
seldom in doubt.
198. The following chest X-ray was recorded
from a patient presenting to an
emergency with breathlessness. What is
the most likely diagnosis?

a) Emphysema

b) Hydropneumothorax

c) Pneumothorax

d) Cardiac tamponade

Correct Answer - B
Ans: B. Hydropneumothorax
The air-fluid level seen on the left side in the given X-ray is
suggestive of a diagnosis of hydropneumothorax.
Hydropneumothorax:
Defined as the presence of both air & fluid within the pleural space
surrounding the lung.
X-ray Findings:
An upright chest X-ray will show air-fluid levels.
Horizontal fluid level is usually well-defined & extends across the
whole length of hemithorax.

click on the box for the Channel link


199. A patient presented with neck pain and
rigidity, which gets relieved after bathing
in hot water and also after exercise.
Cervical X-ray is shown below. What is
the likely diagnosis?

a) Spondylolisthesis

b) Spondylosis

c) Compression fracture

d) Osteoporosis

Correct Answer - A
Ans: A. Spondylolisthesis

Given X-ray showing a forward displacement of the vertebra on
lower one, suggestive of spondylolisthesis.
Spondylolisthesis:
Means forward translation of one segment of the spine upon
another.
Shift is nearly always between L4 and L5, or between L5 and the
sacrum.
Normal discs, laminae and facets constitute a locking mechanism
that prevents each vertebra from moving forwards on the one below.
Forward shift (or slip) occurs only when this mechanism has failed.

click on the box for the Channel link

You might also like